GENERAL PRACTITIONER EXAM
Question Summary
0 of 100 questions completed
Questions:
- 1
- 2
- 3
- 4
- 5
- 6
- 7
- 8
- 9
- 10
- 11
- 12
- 13
- 14
- 15
- 16
- 17
- 18
- 19
- 20
- 21
- 22
- 23
- 24
- 25
- 26
- 27
- 28
- 29
- 30
- 31
- 32
- 33
- 34
- 35
- 36
- 37
- 38
- 39
- 40
- 41
- 42
- 43
- 44
- 45
- 46
- 47
- 48
- 49
- 50
- 51
- 52
- 53
- 54
- 55
- 56
- 57
- 58
- 59
- 60
- 61
- 62
- 63
- 64
- 65
- 66
- 67
- 68
- 69
- 70
- 71
- 72
- 73
- 74
- 75
- 76
- 77
- 78
- 79
- 80
- 81
- 82
- 83
- 84
- 85
- 86
- 87
- 88
- 89
- 90
- 91
- 92
- 93
- 94
- 95
- 96
- 97
- 98
- 99
- 100
Information
Hi, Welcome to General Practioner Exam
You have already completed the Exam before. Hence you can not start it again.
Exam is loading...
You must sign in or sign up to start the Exam.
You have to finish following Exam, to start this Exam:
Results
0 of 100 questions answered correctly
Your time:
Time has elapsed
You have reached 0 of 0 points, (0)
Average score |
|
Your score |
|
Categories
- Not categorized 0%
Pos. | Name | Entered on | Points | Result |
---|---|---|---|---|
Table is loading | ||||
No data available | ||||
- 1
- 2
- 3
- 4
- 5
- 6
- 7
- 8
- 9
- 10
- 11
- 12
- 13
- 14
- 15
- 16
- 17
- 18
- 19
- 20
- 21
- 22
- 23
- 24
- 25
- 26
- 27
- 28
- 29
- 30
- 31
- 32
- 33
- 34
- 35
- 36
- 37
- 38
- 39
- 40
- 41
- 42
- 43
- 44
- 45
- 46
- 47
- 48
- 49
- 50
- 51
- 52
- 53
- 54
- 55
- 56
- 57
- 58
- 59
- 60
- 61
- 62
- 63
- 64
- 65
- 66
- 67
- 68
- 69
- 70
- 71
- 72
- 73
- 74
- 75
- 76
- 77
- 78
- 79
- 80
- 81
- 82
- 83
- 84
- 85
- 86
- 87
- 88
- 89
- 90
- 91
- 92
- 93
- 94
- 95
- 96
- 97
- 98
- 99
- 100
- Unanswered
- Answered
- Review
-
Question 1 of 100
1. Question
1 pointsA male aged 63 years is receiving fluorouracil for colon cancer. Because of his profound nausea, he is prescribed aprepitant. The mechanism of action of this agent is which one of the following?
Correct
Incorrect
Explanation:
Neurokinin A and neurokinin B act on NK1 and NK2 receptors in the CNS and are closely related to substance P. NK1 receptors are present in the brain stem (medulla) centers that control the emetic reflex. Aprepitant (Emend) is an NK1 antagonist that has been approved for chemotherapy-induced nausea and vomiting. Dronabinol (Marinol) is an orally active cannabinoid used for chemotherapy induced nausea and vomiting.
Prochlorperazine (Compazine) is a dopamine antagonist used to treat nausea and vomiting. Benzodiazepines such as lorazepam (Ativan) and diazepam (Valium) bind to GABAA receptors to enhance GABAergic transmission. These agents are useful in anticipatory emesis and in the treatment of vertigo. Scopolamine is an example of a muscarinic antagonist used to treat motion sickness and preoperative nausea. It is not used to treat nausea and vomiting resulting from chemotherapy. -
Question 2 of 100
2. Question
1 pointsLumbar region X-ray of the back of a 67 year old female reveals the presence of a compression fracture. Further examination shows a substantial decrease in the thickness of the other bones of the lower back. If investigations reveal normal levels of calcium, phosphorus, and parathyroid hormone, which drug would most likely worsen her condition?
Correct
Incorrect
Explanation:
Glucocorticoids after calcium metabolism by decreasing the synthesis of the principle proteins of bone matrix by osteoblasts, cause malabsorption of calcium in both the kidneys and gastrointestinal tract, and reduce levels of sex hormones. Osteoporosis is a condition of decreased and/or low bone density that can progress slowly over an extended period of time. If diagnosed early, the fractures associated with the disease can often be prevented. Unfortunately, osteoporosis frequently remains undiagnosed until a fracture occurs. Histologic bone sections generally show thin trabeculae that have normal calcification as well as normal amounts of osteoblasts and osteoclasts. Although calcium can be part of an osteoporosis treatment regimen, patients typically have normal serum levels of calcium. Normal levels of phosphorus, alkaline phosphatase, and parathyroid hormone are often present as well. There are two general classes of osteoporosis: primary and secondary. Primary osteoporosis is the most common type and generally occurs in postmenopausal women secondary to low estrogen levels. Hence, conjugated estrogen replacement therapy is effective in the treatment of these women. Secondary osteoporosis is generally related to a variety of other medical therapies and/or other medical conditions, such as corticosteroid therapy (i.e., cortisone) as well as hyperthyroidism and hypogonadism, respectively. Alendronate, conjugated estrogens, risedronate and vitamin D are all used in the treatment of osteoporosis. Hydrochlorothiazide is a thiazide diuretic used for the treatment of a variety of edematous states and hypertension. Although most diuretics (such as furosemide and torsemide) decrease calcium levels, hydrochlorothiazide increases blood calcium levels. Therefore, this agent will not worsen this patient´s condition. -
Question 3 of 100
3. Question
1 pointsA young mother takes her infant to the pediatrician for the first time. The pediatrician notices the infant´s teeth are yellow. Which antibiotic mother most likely took during pregnancy?
Correct
Incorrect
Explanation:
The teeth mottling occurs when a child is exposed to tetracycline in utero. You should remember that tetracycline is contraindicated in pregnancy and early childhood. Tetracycline is a bacteriostatic drug that binds to the 30s subunit of ribosomes, preventing aminoacyl-tRNA from binding with complementary mRNA. This inhibits peptide bond synthesis. Resistance is plasmid-mediated. Inhibition of peptidyl transferase occurs with chloramphenicol, a broad-spectrum bacteriostatic agent that binds to the 50s subunit of ribosomes. Resistance is plasmid mediated. It has high toxicity (gastrointestinal disturbances, aplastic anemia, and gray baby syndrome), so it is used mainly in severe infections or as a topical agent. Interference with cell wall synthesis occurs with penicillins and cephalosporins, the beta-lactam antibiotics. Resistance to these drugs appears in organisms that have developed beta-lactamases (penicillinases), enzymes that destroy the beta-lactam ring of these medications. The wider spectrum ampicillin, amoxicillin, ticarcillin, and carbenicillin are particularly penicillinase-susceptible. Large, cyclic, lactone-ring structures describe the macrolides: erythromycin, azithromycin, and clarithromycin. They inhibit bacterial protein synthesis by reacting with the 50s ribosomal subunit and preventing the release of the uncharged tRNA. Resistance is plasmid-mediated. Common side effects include gastrointestinal irritation, skin rashes, and eosinophilia. Erythromycin is a popular choice for patients with penicillin hypersensitivity. It is a cytochrome P450 inhibitor and therefore must be used with caution in patients taking other drugs. -
Question 4 of 100
4. Question
1 pointsA new drug´s pharmacokinetic properties are being studied in normal volunteers during phase I clinical trials. The volume of distribution and clearance determined in the first subject are 80 L and 4 L/hr, respectively. The half-life of the drug in this subject is approximately which one of the following?
Correct
Incorrect
Explanation:
The half-life of a drug can be determined using the following equations:
1) CL=k x Vd
4L/h=k x 80L
or
k = 4/80 = 1/20 h-12) t1/2 = 0.7/k
t1/2=0.7 x 20h
t1/2= 14 h -
Question 5 of 100
5. Question
1 pointsIV midazolam is given to a 47 year old woman to supplement the local anesthetics that are injected into her foot during surgery. She suddenly becomes agitated and combative midway through the surgery and exhibits involuntary movements. The anesthesiologist determines that she is having a paradoxical reaction to the midazolam and immediately administers
Correct
Incorrect
Explanation:
Flumazenil is a benzodiazepine antagonist and has been approved to hasten the recovery from benzodiazepines used in anesthetic and diagnostic settings and to reverse the CNS depressant effects following an overdose with benzodiazepines Flumazenil can be used only for benzodiazepines or benzodiazepine receptor agonists, such as zolpidem and zaleplon; it is not useful in reversing the effects of other CNS depressants, such as barbiturates and ethanol.
Glucagon is an antidote for beta-blocker overdose. Naloxone, an opioid receptor antagonist, is an antidote for opioid overdose. Nitrite, or sodium nitrite, is an antidote for cyanide poisoning. Protamine is an antidote for heparin overdose. -
Question 6 of 100
6. Question
1 pointsA 49 year old smoker with deep venous thrombosis is given heparin. Heparin achieves its anticoagulant activity by binding to which one of the following substance?
Correct
Incorrect
Explanation:
Heparin, a highly negatively charged molecule, binds to the coagulation inhibitor antithrombin III, increasing its activity 100 to 1000 times.
Alpha2-antiplasmin binds plasmin. Plasmin functions to break down clots and the binding of alpha2-antiplasmin to plasmin acts to inhibit clot lysis. Alpha2 macroglobulin is a circulating antiprotease macromolecule that assists in preventing blood clotting by inhibiting the action of proteolytic coagulation factors. The remaining answer choices are all clotting factors. They do not interact with heparin. -
Question 7 of 100
7. Question
1 pointsA 34 year old patient with a history of asthma is being treated for symptoms of hypertension. The beta blockers would be an appropriate therapy for this patient is which one of the following?
Correct
Incorrect
Explanation:
If the patient has asthma, you should select a drug that blocks beta1 receptors without affecting the beta2 receptors found in the respiratory smooth muscle. Therefore, a selective beta1 blocker (such as metoprolol) would be appropriate. Isoproterenol is a (nonselective) beta agonist, not antagonist. Labetalol is a mixed alpha and nonselective beta antagonist used for chronic hypertension and hypertensive emergencies. Propranolol is a nonselective beta antagonist. It would block both betas and beta2 receptors and would therefore be contraindicated in a patient with asthma. Timolol is also a nonselective beta antagonist that is used topically in the treatment of glaucoma. -
Question 8 of 100
8. Question
1 pointsA 34 year old SLE patient abruptly stops taking her glucocorticoids because “she is well now and does not want to get fat.” After several days she goes to the ER because she “feels terrible.” The finding would be expected on investigations?
Correct
Incorrect
Explanation:
Long term, particularly daily, use of exogenous systemic corticosteroids causes pituitary/adrenal suppression, and abrupt cessation may induce an Addison like crisis characterized by hypotension, hypoglycemia, and sometimes (if mineralocorticoid function is suppressed as well) hyponatremia and hyperkalemia. Decreased ACTH and low cortisol would also be expected. Treatment is with IV steroids, and, if the patient is in shock, IV saline. The patient should also be examined for other conditions (such as infection) that may have contributed to the acute adrenal crisis. -
Question 9 of 100
9. Question
1 pointsThe mechanism of action of low-molecular-weight heparin is which one of the following?
Correct
Incorrect
Explanation:
The shorter chain low molecular-weight (LMW) fractions of heparin inhibit activated factor X but have less effect on thrombin (and on coagulation in general) than the HMW species. -
Question 10 of 100
10. Question
1 pointsAngina in a 56-year-old man was relieved by buccalglyceryltrinitrate (GTN) 5 mg. His discharge medication was isosorbidemononitrate (ISMN) 20 mg BD. Which factor is responsible for the dose difference between these two formulations?
Correct
Incorrect
Explanation:
Buccal GTN is quickly converted to di- and mono-nitrates which have half- lives of two hours. Its effective duration of action is 30 minutes. It is metabolised by the liver to inorganic nitrite.
ISMN is not subject to first pass metabolism in the liver. The overall half-life of isosorbidemononitrate is about five hours, and that of GTN is two hours. Thus, this would result in a reduced dose of ISMN being required if related to plasma clearance. The sublingual route (which avoids first pass effect and the need for gastrointestinal absorption) is the preferred route for achieving therapeutic effect quickly, as GTN is absorbed efficiently by this route. However, the total dose administered by this route must be limited to avoid excessive effects.
When a much longer duration is needed, oral preparations can be given that contain a sufficient amount of drug to result in sustained systemic blood levels. -
Question 11 of 100
11. Question
1 pointsA 45-year-old man presents with bilateral gynaecomastia. He has been diagnosed with Zollinger-Ellison syndrome in the last year. He underwent normal puberty at age 14. Which of the following drugs would be most likely to cause gynaecomastia?
Correct
Incorrect
Explanation:
The answer to this question is cimetidine which is an H2 receptor antagonist. Blockade of androgen-responsive receptors in the target organ appears to be the most likely mechanism involved. Research has shown that the other drugs listed above which may also be used as part of the treatment of Zollinger-Ellison syndrome have a much lower almost insignificant risk in the development of gynaecomastia.
Other drugs that can cause gynaecomastia include-spironolactone,digoxin, methylodopa, gonadotropins and cyproterone acetate Zollinger-Ellison syndrome The association of peptic ulcer with a gastrin-secreting pancreatic adenoma -50-60% is malignant. It occurs in approx 0.1% of patients with duodenal ulcer disease and is to be suspected in those with multiple peptic ulcers that are resistant to drugs. -
Question 12 of 100
12. Question
1 pointsA 46-year-old male takes Lithium for a bipolar affective disorder. Which of the following drugs would be contra-indicated in conjunction with Lithium?
Correct
Incorrect
Explanation:
Caution should be exercised when taking Lithium and diuretics as the latter may reduce renal clearance of lithium and increase serum lithium concentrations. NSAIDs also increase Lithium concentrations.
Metronidazole, ACEI´s and Calcium channel blockers also increase serum Lithium concentrations. -
Question 13 of 100
13. Question
1 pointsThe best way to avoid benzodiazepine dependence is
Correct
Incorrect
Explanation:
Benzodiazepines have addictive properties. If a benzodiazepine with shorter half- life is used, there is more dependency since the patient will see the effect of the drug wear off faster. The better option is to counsel against long term use of such medicines. When prescribing benzodiazepines the following guidelines should be used:
Do not prescribe benzodiazepines in someone with a history of drug misuse and dependence.
Prescribe the lowest possible doses of benzodiazepines and only prescribe for a few weeks. It is important to remember that patients can get withdrawal symptoms between doses if they are given short- acting benzodiazepines.
Elderly patients are particularly prone to adverse effects of benzodiazepines therefore, need to be careful. -
Question 14 of 100
14. Question
1 pointsWhich of the following micro-organisms is generally sensitive to benzylpenicillin?
Correct
Incorrect
Explanation:
Penicillin binds to specific penicillin-binding proteins (PBP´s) in the cell wall, mainly of gram positive organisms.
Penicillin resistance is usually due to production of altered PBPS or beta-lactamases which leave the…
Penicillin is mainly useful for group A Strep., Group B Strep., meningococcal and pneumococcal infections, though anthrax are also sensitive.
Pneumococci with modified PBPB are an increasing problem -
Question 15 of 100
15. Question
1 pointsA 72-year-old man is treated for chest infection. For past 6 months,he is receiving Warfarin for atrial fibrillation with normal INR. However, his most recent INR was 5.2 (<1.4).Which one of the following drugs is responsible?
Correct
Incorrect
Explanation:
Clarithromycin induces the anticoagulant effect of warfarin, whereas rifampicin would reduce the anticoagulant effect. Ciprof1oxacin and sulphonamides will also increase the anticoagulant effect of warfarin. Temazepam, digoxin and codeine have no appreciable effect.
-
Question 16 of 100
16. Question
1 pointsWhich of the following pharmacological agents acts through the opening of potassium channels?
Correct
Incorrect
Explanation:
Nicorandil is a potent potassium channel activator. It relaxes vascular smooth muscle through membrane hyperpolarization via increased transmembrane potassium conductance and, like nitrates, through an increase in intracellular cyclic GMP. Glibenclamide blocks potassium channels. Amiloride inhibits the action of aldosterone on the distal convolted tubule producing potassium reabsorption. Amiodarone blocks Potassium channels. -
Question 17 of 100
17. Question
1 pointsWhich statement is true regarding gabapentin?
Correct
Incorrect
Explanation:
Gabapentin does not induce cytochrome P450 unlike other anticonvulsants such as phenytoin and phenobarbitone. Gabapentin is a structural analog of GABA, but it does not activate GABA receptors directly.
Vigabatrin may cause visual field defects, which may be irreversible rarely have visual disturbances been associated with gabapentin. Gabapentin is no use in petit mal and is used for add-on therapy in partial or generalized seizures. -
Question 18 of 100
18. Question
1 pointsA 91-year-old man with chronic leukemia presents with gout which his general practitioner treats with Allopurinol. How does Allopurinol prevent the accumulation of uric acid?
Correct
Incorrect
Explanation:
Allopurinol is a xanthine oxidase inhibitor and is converted by this enzyme to alloxanthine in this form it inhibits the conversion of hypoxanthine to xanthine, and the conversion of xanthine to uric acid therefore inhibiting the formation of uric acid. -
Question 19 of 100
19. Question
1 pointsThe best way to avoid benzodiazepine dependence is
Correct
Incorrect
Explanation:
Benzodiazepines have addictive properties. If a benzodiazepine with shorter half- life is used, there is more dependency since the patient will see the effect of the drug wear off faster. The better option is to counsel against long term use of such medicines. When prescribing benzodiazepines the following guidelines should be used:
Do not prescribe benzodiazepines in someone with a history of drug misuse and dependence.
Prescribe the lowest possible doses of benzodiazepines and only prescribe for a few weeks. It is important to remember that patients can get withdrawal symptoms between doses if they are given short- acting benzodiazepines.
Elderly patients are particularly prone to adverse effects of benzodiazepines therefore, need to be careful. -
Question 20 of 100
20. Question
1 pointsThe benzodiazepine which has shortest half life is
Correct
Incorrect
Explanation:
Alprazolam (Xanax) has a half-life of about 12 hours, versus 25 hours for clonazepam and 50 hours for flurazepam, clorazepate, and diazepam. -
Question 21 of 100
21. Question
1 pointsAddition of bupropion (Welbutrin) should be considered in a patient taking a selective serotonin reuptake inhibitor (SSRI) if he develops which one of the following side effects?
Correct
Incorrect
Explanation:
Antidepressants that inhibit serotoneric reuptake have been reported to g interfere with sexual function. Bupropion is a norepinephrine and dopamine reuptake inhibitor with essentially no direct serotonergic activity. Improvement in sexual functioning has been reported when sustained-release bupropion was either substituted for other antidepressants or added to a regimen of SSRIs. Two recent studies have also shown that sustained-release bupropion was well tolerated in the treatment of sexual dysfunction in non-depressed women. -
Question 22 of 100
22. Question
1 pointsA man aged 46 year is diagnosed pulmonary tuberculosis. Which one of the following investigations is essential prior to starting antituberculous therapy?
Correct
Incorrect
Explanation:
Hepatotxicity is a feature of antituberculous treatment. “Liver function should be checked before treatment for clinical cases.” -
Question 23 of 100
23. Question
1 pointsA lady of age 56 years is on treatment for long-standing rheumatoid arthritis. She recently becomes dyspnoeic on mild exertion and developed a dry cough. The oxygen saturation was found to be 87% on air. A diffuse bilateral interstitial infiltrate was seen on chest X-ray. Screening for infection was negative and her symptoms were felt to be drug-induced. This adverse effect is most likely to be caused by which one of the following?
Correct
Incorrect
Explanation:
Methotrexate is a well recognized cause of acute pneumonitis and interstitial lung disease. It is a rare complication of methotrexate therapy but is often fulminant and can be fatal. -
Question 24 of 100
24. Question
1 pointsA type 2 diabetic female of 50 years of age presents with a two month history of painful hands and feet. Diagnosis of sero-positive erosive rheumatoid arthritis is confirmed by investigations. She has some pain relief from NSAIDs. She currently takes metformin 500 mg tds and has good glycaemic control as reflected by an HbA1c of 6.7% (3.8-6.4). Which DMARD would be most appropriate as her initial therapy for early rheumatoid arthritis?
Correct
Incorrect
Explanation:
Guidance recommends the use of DMARDS early in the treatment of Rheumatoid arthritis maintaining function and reducing progression of the disease (SIGN 2001). First line agents include methotrexate and sulphasalazine (SIGN 2000) and most subjects receive Methotrexate. Generally gold is considered more toxic than the former two and hydroxychlorquine is probably less effective. Ciclosporin is again rather more toxic than either methotrexate or sulphasalazine, with nephrotoxicity and immunosuppression and is generally reserved for RhA with systemic features such as vasculitis.
The TNF alpha antagonists, etanercept and infliximab are generally reserved for individuals unresponsive to traditional DMARDS*.
*Ref. Nice and BSR guidelines. -
Question 25 of 100
25. Question
1 pointsA 33 year old man presents with a history and findings that, satisfy DSM-IV criteria for bipolar disorder. Out of the following, which one is the most effective for long-term management of the majority of patients with this disorder?
Correct
Incorrect
Explanation:
Electroconvulsive therapy (ECT) is as effective as medication for the acute treatment of the severe depression and/or mania of bipolar disorder. However, ECT should be reserved for patients with severe mood syndromes who may be unable to wait for mood-stabilizing drugs to take effect. Neuroleptic (antipsychotic) drugs are effective in acute mania, but are not recommended for long-term use because of side effects.
Bipolar depression generally responds to tricyclic antidepressants, SSRIs, and MAO inhibitors, but when used as long-term therapy these drugs may induce episodes of mania.
Anticonvulsants, such as carbamazepine, valproic acid, and benzodiazepines, have been useful adjuncts combined with lithium in patients with breakthrough episodes of mania and/or depression. Lithium ` is the classic mood stabilizer. It has been shown to have antimanic efficacy, prophylactic efficacy in bipolar disorder, and some efficacy in prophylaxis against bipolar depression. Lithium remains the drug of choice for long-term treatment of the majority of patients with bipolar illness. -
Question 26 of 100
26. Question
1 pointsA 27 year old homosexual male is diagnosed with tuberculosis and started on a regimen involving INH. Isoniazid interactions result in which one of the following vitamin deficiency?
Correct
Incorrect
Explanation:
The most common etiology of vitamin B6 (pyridoxine) deficiency results from drug interactions. Most notably, isoniazid, cycloserine, oral contraceptives, and penicillamine are causes. -
Question 27 of 100
27. Question
1 pointsA 55 year old diabetic male is diagnosed with hypertension and prescribed a high dose of an antihypertensive associated with the development of a dry cough and angioedema. An adverse effect most likely to occur with this drug is
Correct
Incorrect
Explanation:
Diabetics with hypertension are usually started on angiotensin converting enzyme (ACE) inhibitors because these drugs are protective against diabetic nephropathy. Examples of ACE inhibitors include enalapril, captopril, and lisinopril. ACE inhibitor hypersensitivity is usually a direct result of alterations in kinin generation in sensitive individuals. These reactions can be life threatening even if they are not true allergic reactions. Therefore, ACE inhibitors are contraindicated in patients with a history of ACE inhibitor induced angioedema, hereditary angioedema, or idiopathic angioedema ACE inhibitors are also associated with the development of a dry, “hacking” cough as well as hyperkalemia. The ACE inhibitors decrease the conversion of angiotensin I to angiotensin II (AII), leading to reduced production of AII, which is a major stimulus for aldosterone secretion. In this fashion, ACE inhibitors markedly reduce the secretion of aldosterone. Aldosterone potently stimulates urinary secretion of potassium; thus, an occasional undesirable side effect of ACE inhibitors is a potentially dangerous elevation of serum potassium. . Decreased secretion of aldosterone is one of the major therapeutic benefits of this class of drugs. Decreased secretion of renin is incorrect, because a reflex increase in renin secretion is expected with ACE inhibitors. Increased total peripheral resistance is incorrect because reduction of circulating angiotensin II causes decreased total peripheral resistance, a major contribution to the therapeutic benefit. Increased urinary excretion of sodium is expected to occur secondary to reduced aldosterone, but this is beneficial because it contributes to the diuretic effect of these drugs. -
Question 28 of 100
28. Question
1 pointsA 70 year old male has a blood pressure of 88/102mm Hg, edema in the scrotal region, and reddish urine. Lab findings are significant for hematuria and moderate proteinuria, as well as red cell casts and white cells in the urine. Additional test reveal abnormal complement levels (C3, C4, CH50) and anti-GBM antibodies as well as abnormal ANCA. Noncytotoxic treatment of this patient is likely to cause which of the following?
Correct
Incorrect
Explanation:
Acute glomerulonephritis is a relatively uncommon cause of acute renal failure, responsible for approximately 5% of all causes of intrinsic renal failure. In general, ANCA-associated and anti GBM associated acute glomerulonephritis can evolve to crescentic glomerulonephritis. The overall clinical outcome is poor unless treatment is started early. Those with acute glomerulonephritis are often hypertensive, edematous, and have an abnormal urinary sediment. The edema generally develops first in areas of the body with low tissue tension such as the periorbital and scrotal regions. Laboratory evaluation typically reveals hematuria, moderate proteinuria (usually < 2 g/d), and cellular elements such as red cells, red cell casts, and white cells. To confirm the etiology of the acute glomerulonephritis, complement levels (C3, C4, CH50), ASO titer, anti-GBM antibody levels, ANCAs, antinuclear antibody titres, cryoglobulins, hepatitis serologies, blood cultures and a renal ultrasound are often undertaken. Treatment measures generally consist of high dose corticosteroids and cytotoxic agents such as cyclophosphamide. The corticosteroids are generally associated with fluid and electrolyte imbalances such as hypernatremia fluid retention hypokalemia and hypocalcemia. Other adverse effects include muscle weakness muscle mass loss gastritis, hyperglycemia the appearance of a Cushingoid state immunosuppression, hypertension, steroid psychosis and osteoporosis. A lupus erythematosus like syndrome is a side effect of hydralazine, procainamide, and quinidine.
-
Question 29 of 100
29. Question
1 pointsA 69 year old female is being evaluated for hip replacement surgery. She has hypertension that is well controlled. She is hospitalized and her cardiac rhythm shows second degree AV block. Which medication is the most likely cause of her arrhythmia?
Correct
Incorrect
Explanation:
Metoprolol is a selective beta 1 adrenergic receptor blocker, which can cause bradycardia and varying degrees of AV block. In this case, the patient has a second degree AV block. Second degree heart block is typically subclassified as follows: Mobitz type I (Wenckebach) AV block (the PR interval progressively lengthens, with the PR interval shortening before the blocked beat) and Mobitz type II AV block (intermittent, nonconducted atrial beats not preceded by lengthening AV conduction). Mobitz type I block may occur as a result of medications, such as beta blockers, digoxin, and calcium channel blockers, or because of increased vagal tone. Mobitz type II block is usually associated with an organic lesion in the conduction pathway. This is less common but more serious, and a pacemaker maybe required. Felodipine is a dihydropyridine calcium channel blocker. It inhibits calcium ion flux into vascular smooth muscle and the myocardium. Unlike verapamil and Diltiazem, which are also calcium channel blockers, it is not associated with bradycardia or AV block. Hydrochlorothiazide is a thiazide diuretic. It inhibits sodium and chloride resorption in the distal convoluted tubule. Terazosin is a peripherally acting alpha l adrenergic receptor blocker. It is used for the treatment of hypertension and benign prostatic hypertrophy. Triamterene is a potassium-sparing diuretic. It inhibits sodium influx through channels in the luminal membrane of the late distal and cortical collecting tubules. Because potassium secretion is coupled with sodium entry in this area, potassium is spared. -
Question 30 of 100
30. Question
1 pointsA 56 year old man has nasal allergies. He also has nocturia and decreased force and caliber of his urinary stream. PSA level is 4.2. Rectal examination shows a diffusely enlarged prostate. What antihistamine would be most appropriate for him?
Correct
Incorrect
Explanation:
The patient is presenting with signs and symptoms highly suggestive of benign prostatic hyperplasia (BPH), which is a common disorder found in men and is often age related. BPH is characterized by a decreased force and caliber of the urinary stream, high postvoid residual urine volume, nocturia, and urinary retention on occasion. The PSA is just above the normal range (0-4 ng/mL); and the physical examination is suggestive of BPH. Anticholinergic agents are generally not used in patients with BPH because of their “anti-SLUDG” properties. These medications decrease salivation, lacrimation, urination, and the ability to defecate. Similarly, these medications act to slow down the gastrointestinal tract by exerting an antagonistic effect on acethylcholine M3 receptors in smooth muscle. However if a patient with BPH had to be prescribed an antihistamine the physician should choose one with low anticholinergic side effects such as cetirizine, fexofenadine or loratadine.
All of these agents produce reversible competitive antagonism of H1 receptors. The antihistaminic effects include inhibition of vascular, gastrointestinal, and respiratory smooth muscle contraction, decreased capillary permeability leading to decreased wheal, flare, and itching response; also anticholinergic effects lead to the “drying up” of mucus secretions. These agents are indicated for the relict of manifestations of immediate type hypersensitivity reactions, and to ameliorate the signs and symptoms of the common cold. -
Question 31 of 100
31. Question
1 pointsA 39 year old worker with constant back pain gets a prescription and takes the recommended drug dose. After 45 minutes, he begins to feel sleepy, has a dry mouth, and can feel his heart “racing.” Which drug was most likely given?
Correct
Incorrect
Explanation:
There are two major therapeutic groups of muscle relaxants: neuromuscular blockers and spasmolytics. Neuromuscular blockers act by interfering with transmission at the neuromuscular end plate, have no CNS activity, and often are used during surgical procedures and in intensive care and emergency medicine to cause paralysis. Spasmolytics, or “centrally acting” muscle relaxants, are used to alleviate musculoskeletal pain and spasms and to reduce spasticity in a variety of neurologic conditions. When patients present with either a muscle spasm or a “strained” muscle, a centrally acting skeletal muscle relaxant is typically prescribed. In addition to these agents, a nonsteroidal anti-inflammatory drug is often added for additional pain control. Although somnolence can be seen with any of the agents listed, xerostomia (dry mouth), mydriasis, and tachycardia are classic anticholinergic side effects. Cyclobenzaprine is a centrally acting skeletal muscle relaxant that is structurally related to tricyclic antidepressants, which are known for their strong anticholinergic side effects. Additional common anticholinergic side effects seen with this agent include blurred vision, urinary retention, and constipation. Less common side effects include agitation, respiratory depression disorientation, tachycardia, and widening of the QRS complex. Baclofen is a centrally acting skeletal muscle relaxant that produces muscle relaxation by inhibition of both monosynaptic and polysynaptic reflexes at the spinal level. This agent is indicated for the treatment of spasticity resulting from multiple sclerosis or secondary to spinal cord injuries; it has also been used in the treatment of trigeminal neuralgia. The most common side effects include transient drowsiness, fatigue, and hypotension. Diclofenac is a nonsteroidal anti-inflammatory drug indicated for the treatment of a variety of disorders associated with pain and inflammation. The most common side effects include dyspepsia, nausea, vomiting, abdominal cramps, and dizziness. Methocarbamol produces muscle relaxation by general CNS depression; it does not have a direct action on the contractile mechanism of striated muscle or nerve fibers. This agent is indicated as an adjunct to rest, physical therapy, and other measures for relief of discomfort in various musculoskeletal conditions. Reported side effects include dizziness, vertigo, ataxia, headache, irritability, bradycardia, hypotension, and syncope. Rofecoxib is a selective cyclooxygenase-2 (CDX-2) inhibitor with anti-inflammatory, analgesic, and antipyretic effects. This agent is used in adults for relief of pain and inflammation caused by osteoarthritis and rheumatoid arthritis, as well other inflammatory conditions. The most common side effects are nausea, vomiting, diarrhea, abdominal distress, flatulence, and anorexia.
-
Question 32 of 100
32. Question
1 pointsA 63 year old man develops premature ventricular contraction (PVCs) while receiving digoxin for atrial fibrillation. He is taking several other medications for multiple problems. The likely cause of this new complication is
Correct
Incorrect
Explanation:
Patients who develop digoxin toxicity gradually during chronic therapy are often hypokalemic and hypomagnesemic secondary to concurrent diuretic therapy. Toxicity can also occur in those with concomitant ventricular dysrhythmias. In this case, the patient´s premature ventricular contractions (PVCs) are most likely related to the development of hypokalemia secondary to the overuse of furosemide, a loop diuretic. All the other agents are associated with the reduction of digoxin blood levels, which would decrease the risk for digoxin toxicity. Colestipol is a bile acid binding resin that is indicated for the treatment of hypercholesteremia. When given with digoxin, this agent will bind in the intestinal tract, which will decrease the absorption of the drug. Glyburide is a sulfonylurea indicated for the treatment of type 2 diabetes. Medications such as glyburide and sulfasalazine, which have sulfa components, are associated with the lowering of serum digoxin levels. Phenytoin is a hydantoin antiepileptic agent indicated for the treatment of tonic clonic and partial complex seizures. This agent increases the metabolism of digoxin, thus lowering digoxin levels. Sulfasalazine is an agent indicated for the treatment of ulcerative colitis and rheumatoid arthritis, and lowers serum digoxin levels. -
Question 33 of 100
33. Question
1 pointsAn elderly patient diagnosed with erectile dysfunction is prescribed a drug that functions by preventing the breakdown of a compound essential for vascular smooth muscle relaxation, allowing increased blood flow to the penis. The enzyme that is involved in the synthesis of this endogenous compound is
Correct
Incorrect
Explanation:
The most common treatment for erectile dysfunction is Sildenafil, when is a phosphodiesterase inhibitor that prevent the breakdown of cGMP, a key signaling intermediate mediating the relaxing effect of nitric oxide on vascular smooth muscles. cGMP is formed from GTP via the enzyme guanviate cyclase. Choline acetyltransferase converts choline into acetylcholine, a neurotransmitter that mediates the synaptic connections of parasympathetic neurons as well as neuromuscular junctions. Cyclooxygenases and lipoxygenases are involved in the synthesis of prostaglandins, thromboxanes, and leukotrienes, which are involved in multiple processes including acute inflammation and blood clot formation. -
Question 34 of 100
34. Question
1 pointsA 62 year old man with congestive heart failure and hypertension has new onset of anginal symptoms during periods of exertion. A calcium channel blocker is given for angina prophylaxis and for his hypertension. The drug that would be most likely to exacerbate the patient´s heart failure is
Correct
Incorrect
Explanation:
Verapamil is a “first generation” calcium channel blocker that has been associated with an accelerated progression of CHF in certain patients. This agent has a strong negative inotropic effect, which leads to a decrease in the force and velocity of myocardial contraction. Hence, this agent would most likely exacerbate the patient heart failure. As a general rule, the use of calcium channel blockers in CHF is reserved for patients who also have hypertension and/or original symptoms. Amlodipine and felodipine are the most; commonly used calcium channel blocking agents in patients with CHF. These two medications may actually produce a small increase in myocardial contractility and cardiac output. Diltiazem is generally avoided in patients with CHF, since it has mild to moderate negative inotropic effects leading to a small decrease in myocardial contractility. However, the negative inotropic effects of verapamil are much greater than those seen with diltiazem. Isradipine is a calcium channel blocker that can safely be used at lower doses in patients with CHF; it seems to have we not effect on myocardial contractility. -
Question 35 of 100
35. Question
1 pointsExamination of a 78 year old type 2 diabetic woman with diabetic nephropathy, and hyperuricemia is remarkable for hypertension. Her blood pressure was also elevated on two prior visits. Which medication would be best for the initial treatment?
Correct
Incorrect
Explanation:
The question is essentially asking, which of the following agents would be recommended in a women with diabetic nephropathy and hyperuricemia? The angiotensin converting enzyme (ACE) inhibitors, such as captopril, are commonly recommended for the treatment of hypertension in diabetic patients, especially those with renal complications, since these agents have been shown to delay the progression of renal disease. Furthermore, these agents are generally well tolerated. Atenolol is a beta-1 adrenergic antagonist that is generally not recommended for diabetic patients since it can “block” the appearance of the normal signs and symptoms of hypoglycemia. The thiazide diuretics, such as hydrochlorothiazide and indapamide, are commonly associated with hyperuricemia. Furthermore, they can precipitate an acute gout attack in patients with hyperuricemia. Thiazides also cause hyperglycemia and hyperlipidemia, which would aggravate similar findings from the patient´s disease. Minoxidil is a direct acting vasodilator most commonly used in the treatment of severe refractive hypertension because of its profound side effect profile. This agent is rarely used as an initial treatment agent. -
Question 36 of 100
36. Question
1 pointsA woman aged 87 years has multiple episodes of syncope and near syncope. She has history of coronary artery disease, asthma, hypothyroidism, diabetes, and hypertension. She states that she often forgets to take her medications, and will take higher doses to compensate for this. ECG reveals third degree block. Which drug overdose caused her arrhythmia?
Correct
Incorrect
Explanation:
Atenolol is a selective beta-1 adrenergic receptor blocker. In high doses it can cause bradycardia and varying degrees of atrioventricular (AV) block. The patient here has a third degree AV block. In third degree AV block, there is no relationship between P waves and QRS complexes. This is also called complete heart block; a pacemaker develops in the ventricles, with both the atria and ventricles beating at the frown rate because action potential from the atria cannot reach the ventricles. Third-degree AV block is most commonly caused by coronary artery disease or degeneration of the cardiac conduction system with agents known to cause AV block, such as presented in this case. The initial management of bradycardia (third-degree AV block in this case) can be treated with atropine, isoproterenol, and dopamine. Albuterol in high doses can cause tachyarrhythmias, not bradyarrhythmias. It is a bronchodilator used in the treatment of asthma and bronchospasm. Glyburide is used in the treatment of type 2 diabetes mellitus. It stimulates the pancreas to release insulin. In high doses, it can cause hypoglycemia. Levothyroxine is synthetic form of thyroid hormone. It is used in the treatment of hypothyroidism. High doses of levothyroxine can cause tachyarrhythmias, not bradyarrhythmias. Montelukast is a leukotriene inhibitor. It is used in the prevention of asthma attacks. It is not associated with cardiac arrhythmias. -
Question 37 of 100
37. Question
1 pointsA 55 year old male´s cecal mass is resected and is found to be an invasive adenocarcinoma, with lymphatic involvement. 5 fluorouracil (5-FU) is added to control metastasis. 5-FU is an analog that substitutes for which compound in DNA synthesis?
Correct
Incorrect
Explanation:
In this question, the only important elements are “cancer chemotherapy,” “5-FU” and “analog in DNA synthesis.” The answer is thymine; 5 -FU stand for 5-fluorouracil, which can be a metabolic analog of either uracil (in RNA, not DNA) or thymine. Substitution of these analogs impairs DNA synthesis of actively proliferating cells, hopefully with a greater effect on rapidly growing tumor cells than on normal cells in the body. Adenine is a purine, rather than pyrimidine, base and is unaffected by 5-FU. Cytosine is a pyrimidine, but is sufficiently different biochemically from uracil and thymine, so as not to be a target. Guanine is a purine, rather than pyrimidine, and is unaffected by 5-FU. Uracil can be replaced by 5-FU, but only occurs in RNA, not DNA. Affects on RNA are bad for the cell, but do not destroy genetic information. -
Question 38 of 100
38. Question
1 pointsA 2 month old infant is brought with acute cardiovascular collapse. His mother gave him drug prescribed to her for salmonella gastroenteritis. Few days later he began to vomit, and began showing an ashen gray skin color. What is the mechanism of drug given to the child?
Correct
Incorrect
Explanation:
The presentation is known as “gray baby syndrome” which results from an infant´s adverse reaction to chloramphenicol. Unlike adults, infants are unable to break down high doses of the drug, and this result in severe cardiovascular collapse and an ashen- gray skin color that is typical of the syndrome. Two mechanisms responsible for chloramphenicol toxicity in neonates include a developmental deficiency of glucuronyl transferase, which is responsible for the hepatic metabolism of chloramphenicol in the first 3 to 4 weeks of life; and an inadequate renal excretion of unconjugated drug. This can occur within 2 to 9 days (usually 4 days). Chloramphenicol is indicated for treatment of typhoid fever and other serious Salmonella infections. It is still used in many underdeveloped nations. Chloramphenicol acts by inhibiting ribosomal peptidyl transferase. Linezolid blocks initiation complex formation and is used for vancomycin resistant infections. Aminoglycosides such as streptomycin and gentamicin act by causing misreading at translation initiation, and are commonly used for treating gram-negative rods. Macrolides such as erythromycin can interfere with translation translocation and are primarily used for gram positive cocci, as well as Chlamydia Mycoplasma Legionella, and Campylobacter, among others. Tetracyclines prevent binding of aminoacyl tRNA to ribosomes, and are useful in treatment of Chlamydia Mycoplasma H. pylori, Rickettsia, Brucella Vibrio and acne. -
Question 39 of 100
39. Question
1 pointsA 59 year old man was previously taking aspirin to reduce pain of rheumatoid arthritis and gastric complications were developing. He is now taking an alternate drug that is relatively similar in potency to aspirin but is less likely to cause gastric irritation. This agent needs to be taken only once a day due to its long half life of 50 hours. Which drug is he now using to treat RA pain?
Correct
Incorrect
Explanation:
When used in recommended doses, piroxicam is similar in potency to aspirin and naproxen for pain relict associated with RA. Piroxicam competitively inhibits both cyclooxygenase (COX) isoenzymes, COX-1 and COX-2, resulting in analgesic, antipyretic, and anti-inflammatory effects. Because of the long half life, this drug needs to be taken only once a day, but it takes 2 weeks to reach therapeutic levels. Another advantage of piroxicam is the low risk of gastrointestinal reactions.
Ibuprofen, ketoprofen, and naproxen all produce dose- dependent gastric irritation. Ibuprofen and ketoprofen need to be taken every 2 hours, where is naproxen can be taken twice daily due to its relatively long half-life of 14 hours. Ketorolac is indicated for acute (<5 days) pain management. -
Question 40 of 100
40. Question
1 pointsA 34 year old woman goes to the physician for a minor outpatient procedure. Physician wants to use a long duration ester for a local anesthetic. The agent that should be used is which one of the following?
Correct
Incorrect
Explanation:
Tetracaine is a long duration ester local anesthetic. Bupivacaine is a long-duration amide. Cocaine is a medium duration ester and an uptake blocker. Lidocaine is a medium duration amide and an antiarrhythmic. Procaine is a short duration ester. -
Question 41 of 100
41. Question
1 pointsA 28 year old drug abuser ingested 15 tablets of 10-mg dextroamphetamine 5 hours ago, and is brought to the emergency department in an agitated state. Which agent can hasten the elimination of the drug from this patient?
Correct
Incorrect
Explanation:
Ammonium chloride acidifies the urine (ascorbic acid could also be used) and hastens the elimination of weak bases, such as amphetamine. This occurs because basic drugs are not charged in their nonionized, free base form, and can easily diffuse out of the urine across tubular epithelial cells, back into the bloodstream. Ionizing the drug (in this case by lowering the urine pH) prevents it from leaving the filtrate, and the drug is excreted in the urine.
Acetazolamide is a carbonic anhydrase inhibitor. It is used to alkalinize the urine in patients with hyperuricemia to increase the solubility of uric acid in the urine. It would tend to delay the elimination of the ingested amphetamine. Penicillamine is effective in chelating lead in cases of poisoning by that metal. It also chelates copper and is used in treatment of Wilson disease. Probenecid blocks renal tubular acid trans-port. It is used as a uricosuric. Sodium bicarbonate would alkalinize the urine, thus delaying the elimination of the ingested amphetamine. -
Question 42 of 100
42. Question
1 pointsA 58 year old man with a history of MI has shortness of breath. On exam, his heart rate is 110/min, respiratory rate is 22/min and has peripheral edema. Auscultation reveals normal sinus rhythm with an S3 gallop. Ejection fraction is reduced. Which drug would alleviate his symptoms by reducing both the preload and afterload on the heart without affecting its isotropic state?
Correct
Incorrect
Explanation:
Everything in this case points to congestive heart failure (CHF): dyspnea, elevated heart rate, S3, peripheral edema, and reduced ejection fraction. A drug that will alleviate the symptoms by decreasing both preload and afterload is necessary. Enalapril is an angiotensin-converting enzyme (ACE) inhibitor. ACE inhibitors (captopril, enalapril, lisinopril) work by blocking the conversion of angiotensin I to angiotensin II in the lungs. Angiotensin II is both a potent vasoconstrictor and a stimulator of aldosterone production. Aldosterone acts by promoting sodium (and thus water) reabsorption by the kidney. An ACE inhibitor will therefore promote vasodilatation (reducing afterload), as well as reduce intravascular volume (decreasing preload). It does not affect the heart´s isotropic state. Although digoxin is a mainstay in the therapy of CHF, it has no effect on either preload or afterload. Instead, it works by having a positive inotropic effect on the heart. The mechanism involves inhibition of Na+/K+ ATPase and a consequent increase in intracellular calcium, which, in turn, increases the heart´s contractile force. Diltiazem is a calcium channel blocker. As their name implies, calcium channel blockers block the influx of calcium (via specific calcium channels) into the cell from the extracellular fluid; this interferes with excitation contraction coupling in cardiac muscle, producing a negative inotropic effect. Although these drugs can be used in the treatment of hypertension and angina (they inhibit the contraction of the vascular smooth muscle and thereby decrease peripheral vascular resistance/afterload), they are contraindicated in patients with CHF because of their negative inotropic effect. Furosemide is a loop diuretic. It decreases intravascular volume (and hence, preload), but at typical doses has no significant effect on either afterload or isotropic state. Diuretics are often used in the treatment of CHF, and furosemide is used for the immediate relief of pulmonary congestion. Propranolol is a beta blocker. It has a negative inotropic effect on the heart and is therefore contraindicated in patients with CHF. -
Question 43 of 100
43. Question
1 pointsA male aged 72 years with prostate cancer is treated with leuprolide. The mechanism of action of this drug is which one of the following?
Correct
Incorrect
Explanation:
Leuprolide is a GnRH analog. Given longterm in a continuous fashion, it will inhibit FSH and LH release, thereby decreasing testosterone production and exacting a chemical castration in men. It can be used in the treatment of prostate cancer, polycystic ovary syndrome, uterine fibroids, and endometriosis. Inhibition of 5α-reductase is the mechanism of action of finasteride. It thereby inhibits the production of dihydrotestosterone. It is used in the treatment of benign prostatic hyperplasia (BPH). Flutamide is another drug used in the treatment of prostate cancer. It is a competitive antagonist at androgen receptors. Since LH activates interstitial cells to secrete testosterone, a synthetic analog of LH would not be appropriate treatment for prostatic cancer. The same goes for a testosterone analog. -
Question 44 of 100
44. Question
1 pointsMetabolic alkalosis is associated with?
Correct
Incorrect
Explanation:
Diuretic therapy – loop diuretics and thiazides can both initially cause increase in chloride, but once stores are depleted, urine excretion will be below < 25 mEq/L. The loss of fluid from sodium excretion causes a contraction alkalosis.
Mineralocorticoid excess (Conn´s syndrome) is also associated with a metabolic alkalosis.
Metformin causes metabolic acidosis.
Salicylates are themselves acidic and produce a metabolic acidosis. Normal saline has a pH of 5 and may produce a mild metabolic acidosis with significant infusions. -
Question 45 of 100
45. Question
1 pointsA 63-year-old male has Benign Prostatic hyperplasia and he is commenced on Finasteride. Through which of the following mechanisms does Finasteride function?
Correct
Incorrect
Explanation:
Finasteride is a 5-alpha-reductase inhibitor preventing the conversion of testosterone to the active dihydrotestosterone (DHT).
Consequently this agent opposes testosterone, so gynaecomastia and reduced libido are common side effects.
It is also taken orally (under the brand name Propecial) for the treatment of male pattern hair loss! -
Question 46 of 100
46. Question
1 pointsWhich one of the following is not associated with manic behavior?
Correct
Incorrect
Explanation:
Mania can occur by chance association during drug treatment, particularly in patients predisposed to mood disorder. Drugs with a definite propensity to cause manic symptoms include levodopa, corticosteroids and anabolic-androgenic steroids. Antidepressants of the tricyclic and monoamine oxidase inhibitor classes can induce mania in patients with pre-existing bipolar affective disorder.
Drugs which are probably capable of inducing mania, but for which the evidence is less scientifically secure, include other dopaminergic anti-Parkinsonian drugs, thyroxine, iproniazid and isoniazid, sympathomimetic drugs, chloroquine, cimetidine, baclofen, alprazolam, captopril, amphetamine and phencyclidine. -
Question 47 of 100
47. Question
1 pointsAn 85 year old woman nursing-home resident with a primary diagnosis of Alzheimer´s disease develops a pattern of behavioral symptoms consisting of significant verbal and physical aggression toward stag and other residents on at least three occasions. She has also experienced hallucinations that have been very stressful to her. After other causes have been excluded, you decide to treat her abnormal behavior with a medicine. The most appropriate choice would be
Correct
Incorrect
Explanation:
Secondary causes of behavioral symptoms include adverse medication effects, metabolic causes, infections, dehydration, pain, delirium, fecal impaction, and injury. Appropriate first-line pharmacologic treatment of nursing-home residents, who have severe behavioral symptoms with psychotic features, such as hallucinations and delusions that are causing distress, consists of atypical antipsychotics such as risperidone. The other choices listed are not atypical antipsychotics. -
Question 48 of 100
48. Question
1 pointsA 34 year old woman presents with restlessness, nausea and unstable vital signs. She has a history of depression. She was recently switched from sertraline (SSRI) to phenelzine (MAOI). On examination she is tachycardic and tachypneic and her blood pressure is 140/90 mmHg. Her neurological examination is normal aside from hyperreflexia and myoclonous. What is the most likely diagnosis?
Correct
Incorrect
Explanation:
Serotonin syndrome is a condition characterized by dangerously high levels of serotonin in the body. It occurs when you take certain combinations of prescription medications that affect serotonin levels in the body. For example, serotonin syndrome can occur if you take selective serotonin reuptake inhibitors (SSRIs) and monoamine oxidase inhibitors (MAOIs). Signs and symptoms of serotonin syndrome range from restlessness and rapid heartbeat to muscle rigidity and seizures. These go away quickly with treatment, which may include discontinuing use of the medications causing the symptoms along with taking annex drugs such as muscle relaxants and serotonin production blocking agents. If not treated quickly, serotonin syndrome can become life-threatening.
Serotonin syndrome can occur when you start a single new drug that affects serotonin levels or when you take an increased dose of such a drug that you´ve previously used. -
Question 49 of 100
49. Question
1 pointsWith regard to poisoning /overdose:
Correct
Incorrect
Explanation:
Aspirin causes hyperventilation which may result in a respiratory alkalosis; massive overdose may cause a metabolic acidosis. Phenobarbitone and chlormethiazole both suppress the central nervous system causing hypoventilation, hypotension and hypothermia. Ethylene glycol causes a metabolic acidosis.
Methanol is metabolized to formaldehyde and formic acid. -
Question 50 of 100
50. Question
1 pointsThe most helpful medication in a bulimic patient is
Correct
Incorrect
Explanation:
Bulimia nervosa is recurrent episodes of binge eating followed by self- induced vomiting, laxative or diuretic abuse, vigorous exercise, or fasting.
Most symptoms and physical complications result from purging. Self- induced vomiting leads to erosion of dental enamel of the front teeth and to painless salivary gland enlargement. Serious fluid and electrolyte disturbances, especially hypokalemia, occur occasionally.
The diagnosis is suspected when patients express marked concern about weight gain and have wide fluctuations in weight, especially with excessive laxative use or unexplained hypokalemia. Swollen parotid glands, scars on the knuckles (from induced vomiting), and dental erosion are danger signs.
Treatment is with psychotherapy and SSRIs, especially fluoxetine. -
Question 51 of 100
51. Question
1 pointsA 62 year old woman suffering from rheumatoid arthritis has been on long term therapy to control her disease. She now presents to you with history of increasing shortness of breath and a chest radiograph reveals “bilateral interstitial shadowing”. Investigations for infections were negative. Her symptoms are caused by which one of the following medications?
Correct
Incorrect
Explanation:
Methotrexate is a recognized cause of pulmonary fibrosis. However, it is sometimes used in the treatment of idiopathic pulmonary fibrosis as a steroid sparing agent. “Pulmonary parenchymal or pleural reactions to chemotherapeutic agaents used in the management of patients with malignant disease are being recognized with increasing frequency. Alkylating agents, asparaginase, bleomycin, methotrexate and procarbazine have all been implicated” West J Med. 1977 October; 127(4): 292298. “Drug-related interstitial pneumonia should also be considered in rheumatoid arthritis patients on methotrexate or newer drugs such as lerlunomide. “Curr Opin Pulm Med 2006 Sep; 12 (5): 346-53. -
Question 52 of 100
52. Question
1 pointsA female of 80 years of age presents with bilaterally painful knees. There was no history of gastrointestinal diseases. Crepitus was present on examination but had a full range of movement of both knees. The initial therapy that is most appropriate for her painful knees is:
Correct
Incorrect
Explanation:
This women has osteoarthritis (OA) of the knees. The principle goal of systemic therapy is to provide the most effective pain relief with the least associated toxicity. Paracetamol is the initial therapy recommended for the treatment of OA of the hip and knee. Studies have shown that the short-term and long-term efficacy of paracetamol is comparable with that of ibuprofen and naproxen in people with knee osteoarthritis. Specific COX-2 inhibitors such as celecoxib have clinical benefit similar to that of traditional NSAIDS, but less GI toxicity although issues remain regarding their cardiovascular risk. They may be used in patients with GI intolerance of traditional NSAIDs. -
Question 53 of 100
53. Question
1 pointsA 45 year old male admitted to the psychiatric ward is given haloperidol. He then develops restlessness. Constantly gets up, then sits back down, and cannot sit still for any period of time. Which of the following side effects of haloperidol has the patient developed?
Correct
Incorrect
Explanation:
The side effects of antipsychotic medications are as follows: Parkinsonism: involves tremor, `increased muscle tone, bradykinesia or akinesia, drooling, postural instability, loss of spontaneity, micrographia, and seborrhea.
Akathisia: motor restlessness accompanied by a feeling of inner restlessness (sometimes described as anxiety), an urge to move, and an inability to sit still, patients may pace or continuously move their feet and legs. Dystonia: sustained contraction of muscles, twisting postures and may be visible only when the patient is moving. Neck and arm muscles are most commonly affected. Tardive dyskinesia: hyperkinesia, involuntary body movements most often of the muscles of the face, mouth, and tongue. -
Question 54 of 100
54. Question
1 pointsWhich of the following medications may cause amnesia as a side effect?
Correct
Incorrect
Explanation:
Triazolam is a benzodiazepine medications used for sleep disorders. It can be addictive if used improperly. This medication can cause sedation and, because of this, use of triazolam by seniors can contribute to falls and accidents and cause confusion and amnesia. -
Question 55 of 100
55. Question
1 pointsPatients who take digitalis are at increased risk for arrhythmias if they develop
Correct
Incorrect
Explanation:
Patients who receive digitalis and develop hypokalemia are at greater risk for arrhythmias than those who do not take digitalis. The other metabolic abnormalities are not associated with a greater risk for arrhythmias in those who take digitalis. -
Question 56 of 100
56. Question
1 pointsThe pharmacologic agents that is most useful for treatment of generalized anxiety disorder is
Correct
Incorrect
Explanation:
Propranolol is the pharmacologic agent of choice for single event phobias. Buspirone, an anxiolytic agent, is utilized for the treatment of generalized anxiety disorder. Fluoxetine, or Prozac, is used for the treatment of obsessive compulsive disorder. Imipramine is a tricyclic anti depressant which is used for the treatment of panic disorder. Lorazepam is a benzodiazepine which is used for the treatment of post-traumatic stress disorder. -
Question 57 of 100
57. Question
1 pointsThe drug of choice in the acute management of acetaminophen overdose is
Correct
Incorrect
Explanation:
N-acetylcysteine or Mucomist is very beneficial when administered promptly to individuals with acute acetaminophen overdoses. Naloxone is a narcotic analogue that is not helpful for acetaminophen. Induction of vomiting with ipecac is not beneficial. The chelating agent, British anti Lewisite (BAL) is not helpful, nor is charcoal. -
Question 58 of 100
58. Question
1 pointsBronchospasm is most likely caused by which one of the following antihypertensive agent?
Correct
Incorrect
Explanation:
Captopril, an angiotensin converting enzyme inhibitor (ACE inhibitor) can reduce renal perfusion and thus cause hyperkalemia. Lasix, a potent loop diuretic, can increase renal losses and thus cause hypokalemia. Hydralazine can produce arthralgias, fever, and serum sickness similar to systemic lupus erythematosus. Propranolol, a beta-blocker, can produce bronchospasm. -
Question 59 of 100
59. Question
1 pointsResistance of bacteria to aminoglycosides CANNOT involve which one of the following mechanisms?
Correct
Incorrect
Explanation:
Alteration of porins in Gram negative bacteria influences the susceptibility to cephalosporin antibiotics, but not to aminoglycosides. Aminoglycosides are taken up by an energy dependent transport process, which is modified in some resistant organisms. Modification of aminoglycosides by enzymatic phosphorylation or adenylation inactivates many of these drugs and renders the organism resistant. Modification of the ribosomal binding protein of aminoglycosides was one of the first mechanisms of antibiotic resistance to be characterized. -
Question 60 of 100
60. Question
1 pointsA 60 year old man has altered mental status and an enlarged abdomen. Investigations reveal elevated ALT and AST as well as increased blood ammonia levels. A drug is administered to lower his blood ammonia levels. The mechanism of action of the medication he was most likely administered is
Correct
Incorrect
Explanation:
This patient has hepatic encephalopathy. Severe liver disease impairs the capacity to detoxify ammonia generated in the colonic bacterial metabolism of urea. Normal bacterial flora degrade lactulose into lactic acid, formic acid, and acetic acid. This acidic environment ionizes ammonia in the colon to the ammonium ion. Once ammonia is ionized into the ammonium ion, it cannot diffuse across the colon membrane and is ultimately excreted in the stool. This therapy significantly lowers blood ammonia levels. Lactulose also can inhibit nonbacterial, glutamine dependent ammonia production in the intestinal wall. α2-adrenergic agonists such as Clonidine are primarily used to treat hypertension because of its action to decrease sympathetic nervous system tone. However, Clonidine can also stimulate α2-adrenergic receptors in the enteric nervous system (ENS) and decrease the parasympathetic tone to the GI tract. Cholestyramine is an example of a bile acid sequestrant used to lower LDL cholesterol. Chelators are used to treat heavy metal toxicity. One example of a chelator is penicillamine, which is used to lower copper levels in patients with Wilson disease. Opioid receptor agonists are used to treat diarrhea and pain. An example of an opioid receptor agonist is diphenoxylate. -
Question 61 of 100
61. Question
1 pointsA 31 year old female has a BMI of 35. She has been trying exercise regularly and eat smaller portion, but has not beenLOSING WEIGHT
successfully. A drug is prescribed to reduce her appetite. What drug was likely prescribed?
Correct
Incorrect
Explanation:
Sibutramine (Meridia) is a monoamine oxidase inhibitor that also blocks serotonin and norepinephrine reuptake. It is prescribed to reduce appetite and may also increase energy expenditure. Megestrol is a progesterone derivative that increases appetite, and would therefore be contraindicated in this patient. Metoclopramide is a dopamine antagonist used as an antiemetic. It acts in the chemoreceptor trigger zone, and also increases gastrointestinal motility and promotes gastric emptying. Misoprostol is a prostaglandin E1 agonist that acts on parietal cells to decrease gastric acid secretions. Octreotide is a synthetic octapeptide somatostatin analog used to treat variceal bleeding. It is also used to treat diarrhea in a variety of endocrine disorders such as carcinoid, gastrinoma, and glucagonoma. -
Question 62 of 100
62. Question
1 pointsIn the ER a 47 year old man presents with diffuse muscle weakness and palpitations. On examination tachycardia, mydriasis, and elevated body temperature are noted. All of this patient´s symptoms are most likely caused by
Correct
Incorrect
Explanation:
Botulism is caused by a protein toxin produced by the gram-positive, spore forming and obligate anaerobe Clostridium botulinum. The botulinum toxin selectively cleaves the docking proteins for ACh exocytosis. As a result, there is decreased ACh transmission in all parasympathetic neuroeffector junctions as well as decreased sympathetic cholinergic transmission in sweat glands and neuromuscular junctions. Tachycardia, mydriasis, and elevated body temperature are a result of decreased ACh activity on muscarinic receptors in parasympathetic neuroeffector junctions and sympathetic cholinergic innervation of sweat glands. Skeletal muscle weakness results from decreased ACh stimulation of nicotinic receptors in neuromuscular junctions. Atropine, a muscarinic receptor antagonist causes tachycardia, mydriasis, and elevated body temperature but does not cause muscle weakness. Ephedrine is an indirect sympathomimetic and would cause tachycardia, mydriasis, and possibly elevated body temperature from increased metabolic rate. However, it would not cause diffuse muscle weakness.
-
Question 63 of 100
63. Question
1 pointsWhich one of the following is most effective drug in reversing the paralysis of vecuronium?
Correct
Incorrect
Explanation:
Vecuronium is a non-depolarizing neuromuscular blocking agent. Vecuronium antagonizes the acetylcholine receptors at the neuromuscular junction and thus reversal is accomplished by administration of agents that increase cholinergic activity. Such agents include neostigmine, edrophonium, and other cholinesterase inhibitors. Atracurium will add to the patient´s paralysis as it is also a non-depolarizing neuromuscular blocking agent.
Dantrolene prevents the release of calcium from the sarcoplasmic reticulum of skeletal muscle. Thus, anesthesiologists use this medication to treat malignant hyperthermia associated with the concomitant use of succinylcholine and halothane. It is also used to treat neuroleptic malignant; syndrome. It will not reverse the paralysis of a non-depolarizing neuromuscular blocking agent.
Succinylcholine will not reverse the paralysis of a non-depolarizing agent. Succinylcholine is a depolarizing neuromuscular junction blocker that has two phases of action. In Phase 1, prolonged depolarization, the paralysis cannot be reversed. Administration of cholinesterase inhibitors that would reverse non-depolarizing agents only potentiates a depolarizing agent´s effects during this phase. In Phase II, the neuromuscular junction is repolarized but blocked. In this phase, cholinesterase inhibitors will reverse the blockade. Time alone will reverse the block as the drug is metabolized, but the reversal can be hastened with other medication. -
Question 64 of 100
64. Question
1 pointsA study is going on a new drug for the treatment of estrogen-sensitive breast cancer. Clinical trials show that it increases plasma levels of LH and FSH and decreases plasma estradiol levels. It also decreases endometrial proliferation. Which one of the following is most likely blocked by this drug?
Correct
Incorrect
Explanation:
Inhibition of aromatase decreases the synthesis of estradiol from testosterone. As a result of decreased estradiol, there is less stimulation of estrogen receptors on the breast. Decreased stimulation of estrogen receptors in the hypothalamus and anterior pituitary results in decreased negative feedback and an increase in the gonadotropins, LH and FSH. Also decreased estrogen receptor stimulation of the endometrium results in less proliferation.
Blocking androgen receptors would not cause the effects described above. Blocking estrogen receptors on the breast would help treat estrogen sensitive breast cancers. There would also be less negative feedback suppression of gonadotropins. Therefore, blocking estrogen receptors would increase circulating gonadotropins and estradiol. Blocking estrogen receptors on the uterus would decrease endometrial proliferation. Blocking 5 alpha-reductase, the enzyme responsible for converting testosterone to dihydrotestosterone (DHT), and blocking progesterone receptors would not have any of the effects described above. -
Question 65 of 100
65. Question
1 pointsA 45 year old man is admitted to the hospital emergency room after MI. Investigations reveal a two-fold elevation of his LDL cholesterol. Lovastatin is prescribed. It will inhibit which of the following enzymes?
Correct
Incorrect
Explanation:
Hydroxymethyl glutaryl-CoA reductase (HMG-CoA reductase) catalyzes the rate-limiting step in cholesterol synthesis, in which HMG-CoA (formed from 3 acetyl-COA molecules) is reduced to mevalonic acid, using 2 NADPH. Lovastatin and pravastatin reduce cholesterol synthesis and lower serum cholesterol levels by inhibiting this enzyme. Lovastatin is metabolized by the CYP3A isoform of cytochrome P450. In contrast, gemfibrozil and clofibrate lower cholesterol by increasing the activity of lipoprotein lipase which is produced by the endothelial cells of the vasculature of adipose and muscle tissue. Acetyl-CoA carboxylase catalyzes the first step in fatty acid synthesis. Carbamoyl phosphate synthetase catalyzes the rate-limiting step in urea synthesis. Pyruvate dehydrogenase catalyzes the transition step between glycolysis and the TCA (Krebs) cycle. Uridyl transferase is employed in galactose metabolism. -
Question 66 of 100
66. Question
1 pointsA 58 year old man is given antibiotic therapy for Pseudomonas aeruginosa with ceftazidime and gentamicin. Assuming that the half live of gentamicin is 3 hours in this patient, what percentage of the initial dose will most likely remain in his body after 6 hours?
Correct
Incorrect
Explanation:
The half life of a medication is the time required for one half the total amount of the medication to be excreted from the body. For any given first order elimination process, 50% the remaining amount of drug is eliminated at the end of the first half life, 50% of the remaining amount, of drug in the body is eliminated at the end of the second half-live, and soon. Therefore, after 3 hours 50% of the gentamicin will be remaining in the bony. After another 3 hours (6 hours total), 50% of the remaining 50% will be eliminated. In other words, after 6 hours 75% of the gentamicin will be eliminated, and 25% of the initial dose will remain. -
Question 67 of 100
67. Question
1 pointsA patient is receiving Adriamycin, bleomycin, vinblastine, and dacarbazine (ABVD) for Hodgkin disease. A drug is prescribed that blocks dopamine receptors to counteract a side effect of the ABVD regimen. The drug that was most likely prescribed is which one of the following?
Correct
Incorrect
Explanation:
Prochlorperazine (Compazine) is a phenothiazine that blocks dopamine receptors in the chemoreceptor trigger zone, which is located in the floor of the fourth ventricle in the medulla. It is a commonly used antiemetic. Dronabinol is a cannabinoid that is taken orally and is used as an antiemetic to control nausea induced by chemotherapy. Granisetron is a 5-HT3 antagonist that is also used as an antiemetic to control nausea induced by chemotherapy. Meclizine is an H1 antagonist used to treat motion sickness and true vertigo. Megestrol is a progestational agent that is used to stimulate appetite; it is not an antiemetic. -
Question 68 of 100
68. Question
1 pointsA 71 year old man presents with fatigue and exertional and nocturnal dyspnea. Examination is remarkable for rales and a gallop rhythm. CXR reveals cardiac enlargement. Which drug would delay the progress of his disorder?
Correct
Incorrect
Explanation:
The patient is presenting with the classic signs and symptoms of congestive heart failure (CHF). Heart failure is a pathologic state in which an abnormality of cardiac function produces a failure of the heart to pump blood throughout the body at a rate sufficient to meet the requirements of the metabolizing tissues. Some of the adaptive mechanisms to compensate for the “failing heart” are increasing pre-load (through the Frank Starling mechanism), development of myocardial hypertrophy (to restore the increased ventricular wall stress to within normal limits) redistribution of cardiac output from non-vital organs to vital organs, and neurohumoral adjustments. The major criteria for diagnosing heart failure are the appearance of paroxysmal nocturnal dyspnea, neck vein distension, rales, cardiomegaly, acute pulmonary edema, S3 gallop, increased venous pressure (> 16 cm H2 O), and positive hepatojugular reflex. Of the agents listed, only captopril is proven to delay the progression of this condition by protecting the ventricles from remodeling. Captopril is an ACE inhibitor that is now considered to be a standard therapy for the treatment of CHF. In addition to the protective effects, this agent causes vasodilation and blocks the detrimental neurohormonal activity´ associated with the disorder. None of the other agents have been proven to protect the failing heart in this condition. Digoxin is an agent that produces a positive inotropic effect by increasing the force and velocity of myocardial contraction. Furosemide is also considered to be a first-line agent in the treatment of heart failure. This agent is indicated for the treatment of edema associated with CHF, hepatic cirrhosis, and renal disease, as well as treatment of hypertension (furosemide and torsemide). Hydrochlorothiazide is used in the treatment of heart failure. This agent is indicated for the treatment of edema associated with CHF as well as treatment of hypertension. Verapamil is a calcium channel blocker indicated for the treatment of hypertension and a variety of cardiac arrhythmias. This agent is contraindicated in the treatment of heart failure, since it has a strong negative inotropic effect on the heart. -
Question 69 of 100
69. Question
1 pointsAn elderly patient resident of a nursing home is diagnosed with Streptococcus pneumoniae pneumonia. She is treated with penicillin. Mechanism of action of this antibiotic is which one of the following?
Correct
Incorrect
Explanation:
Peptidoglycan is a heteropolymeric component of the bacterial cell wall that provides mechanical stability secondary to its highly crosslinked latticework structure. The biosynthesis of the peptidoglycan may be broken down into three stages. The third stage of this process involves completion of the crosslink, which is accomplished by a transpeptidation reaction occurring outside the cell membrane. This third step in peptidoglycan synthesis is inhibited by the beta-lactam antibiotics, such as penicillin. Penicillin acts by inhibiting transpeptidation reactions that are necessary to form the cross-linkages in the peptidoglycan (cell wall) material. The enzymes that are targeted by penicillin are carboxypeptidases and transpeptidases that are found as transmembrane proteins in the cytoplasmic membrane. Teichoic acids of a gram-positive organism function in attachment to the body surfaces. This is not the site of action of penicillin. Capsular polysaccharide of a bacterium is not the site of action of penicillin. Cell wall of a bacterium is made of peptidoglycan. Although destruction of peptidoglycan is the ultimate result of treatment with penicillin, it is not the site of action of the drug. The lipids of the cell membrane are not targeted by penicillin. -
Question 70 of 100
70. Question
1 pointsA patient is administered a continuous infusion (IV) of morphine at a dose of 0.1 mg/min in the ER. Half life of morphine is 1.9 hours and volume of distribution (Vd) is 230 L. The clearance (CL) for morphine is 60 L/h. What is the expected steady-state concentration that will be attained?
Correct
Incorrect
Explanation:
In the calculation of an appropriate dosing rate, drug clearance is the most important pharmacokinetic value to consider when determining a rational steady state drug dosage regimen. At steady-state, the dosing rate (“rate in”) must equal the rate of elimination (“rate out”).
Dosing rate = Css x CL. The equation can be rearranged to solve for Css such that Css = Dosing rate/CL = (0.1 mg/min)/1 L/min =0.1 mg/L. -
Question 71 of 100
71. Question
1 pointsA female aged 23 years presents with complaints of depressed mood and insomnia for the past two years. Amitriptyline is prescribed. This agent is believed to exert its antidepressant action by which mechanism?
Correct
Incorrect
Explanation:
Tricyclic antidepressants, such as amitriptyline, nortriptyline, and doxepin, possess three primary pharmacologic actions in varying degrees. These agents inhibit biogenic amine (norepinephrine and possibly serotonin) reuptake at the presynaptic neuron, (2) induce sedation, and (3) produce central and peripheral anticholinergic actions. The antidepressant effects are related to the inhibition of biogenic amine reuptake at the presynaptic neuron. Tricyclic antidepressants are indicated for the treatment of clinical depression, with or without anxiety and/or sleep disturbances. Medications that act as agonists at dopamine receptors are used in the treatment of Parkinson disease. Pharmacologic agents such as propranolol and metoprolol are known as beta-blockers, since they block beta-adrenergic receptors. Beta-blockers are indicated for the treatment of hypertension, angina pectoris, and various other cardiovascular conditions. Amphetamines act by increasing the release of norepinephrine from central noradrenergic neurons. They are indicated for the treatment of narcolepsy, attention deficit disorder with hyperactivity, and exogenous obesity.
-
Question 72 of 100
72. Question
1 pointsA 54 year old male has tingling in his feet and recurrent blurry vision. He is obese, rarely exercises and eats an excessive amount of fatty, high caloric food. He takes no medications. A fasting plasma glucose level is 169 mg/dL on this visit and 172mg/dL on a subsequent visit. Which drug used in his condition´s treatment has no effect on the secretion of insulin?
Correct
Incorrect
Explanation:
Metformin is often used in conjunction with oral hypoglycemic agents for the treatment of type 2 diabetes. Its mechanism of action is twofold: (1) it decreases the production of glucose in the liver; and (2) it increases the uptake of glucose in the liver. Metformin has no effect on the secretion of pancreatic insulin. Acetohexamide, chlorpropamide, glyburide, and tolbutamide are oral hypoglycemic agents that are sulfonylurea derivatives. These agents stimulate secretion of insulin from the pancreas. -
Question 73 of 100
73. Question
1 pointsAn 11 year old asthmatic is prescribed a cromolyn sodium inhaler to be administered prior to vigorous activity to prevent an attack. The mechanism of action of this drug is which one of the following?
Correct
Incorrect
Explanation:
Cromolyn sodium inhibits the degranulation of mast cells, thereby preventing the release of histamine and other bronchoconstrictive autacoids. It is administered by inhalation and is used prophylactically to prevent an asthma attack (especially in children). It has no direct bronchodilator, adrenergic, antihistaminic, or anti-inflammatory actions. The remaining answer choices relate to some of the other major categories of drugs available for the treatment of asthma: Muscarinic antagonists work by inhibiting acetylcholine induced bronchoconstriction. Atropine and other belladonna alkaloids used to be first line drugs for the treatment of asthma, but with little success. The newer agent, ipratropium, is a quaternary amine designed for aerosol use. It is a much more successful agent than the earlier muscarinic blockers and has limited side effects because it is minimally absorbed. Corticosteroids are useful in asthma because they reduce inflammation and edema. They also potentiate the bronchodilating effects of the adrenergic agonists. Their use is limited by their many systemic side effects, and they are typically used only in cases of severe or acute bronchospasm or in the treatment of status asthmaticus. Fewer systemic side effects occur in aerosolized preparations such as beclomethasone dipropionate. Selective beta2 agonists include metaproterenol, albuterol, and terbutaline. These drugs are the mainstay in the treatment of acute asthma. They work by stimulating beta 2 mediated bronchodilation (bronchial smooth muscle relaxation) without the cardiac side effects of nonselective beta agonists, such as isoproterenol. Note that new long acting beta agonists have been developed (salmeterol and formoterol) for prophylactic use. -
Question 74 of 100
74. Question
1 pointsAngina in a 56-year-old man was relieved by buccalglyceryltrinitrate (GTN) 5 mg. His discharge medication was isosorbidemononitrate (ISMN) 20 mg BD. Which factor is responsible for the dose difference between these two formulations?
Correct
Incorrect
Explanation:
Buccal GTN is quickly converted to di- and mono-nitrates which have half- lives of two hours. Its effective duration of action is 30 minutes. It is metabolised by the liver to inorganic nitrite.
ISMN is not subject to first pass metabolism in the liver. The overall half-life of isosorbidemononitrate is about five hours, and that of GTN is two hours. Thus, this would result in a reduced dose of ISMN being required if related to plasma clearance. The sublingual route (which avoids first pass effect and the need for gastrointestinal absorption) is the preferred route for achieving therapeutic effect quickly, as GTN is absorbed efficiently by this route. However, the total dose administered by this route must be limited to avoid excessive effects.
When a much longer duration is needed, oral preparations can be given that contain a sufficient amount of drug to result in sustained systemic blood levels. -
Question 75 of 100
75. Question
1 pointsOut of the following, which drug would cause most concern in an overdose?
Correct
Incorrect
Explanation:
Tricyclic antidepressant overdose is a significant cause of fatal drug poisoning. The severe morbidity and mortality associated with these drugs is well documented and due to their cardiovascular and neurological toxicity. Additionally, they are a serious problem in the pediatric population due to their inherent toxicity and the availability of these in the home when prescribed for bed wetting and depression.
The central nervous system and heart are the two main systems that are affected. Initial or mild symptoms include drowsiness, a dry mouth, nausea, and vomiting. More severe complications, include hypotension, cardiac rhythm disturbances, hallucinations, and seizures. Electrocardiogram (ECG) abnormalities are frequent and a wide variety of cardiac dysrhythmias can occur, the most common being sinus tachycardia and intraventricular conduction delay (QRS prolongation). Seizures and cardiac dysrhythmias are the most important life´ threatening complications. -
Question 76 of 100
76. Question
1 pointsAll of the following are symptoms of barbiturate withdrawal, except
Correct
Incorrect
Explanation:
Barbiturates are prescribed as anticonvulsants, sedatives, and general anesthetics. They can also mimic some of the characteristics of alcohol intoxication (including euphoria, elation, and uninhibited behavior), which make them candidates for abuse. These drugs depress the respiratory and nervous system functions, and because abusers rapidly build up a tolerance to the effects of the drug, fatal overdose or coma can easily occur.
Symptoms of withdrawal syndrome appear 12-20 hours after the last dose, they include anxiety, irritability, elevated heart and respiration rate, muscle pain, nausea, tremors, hallucinations, confusion, and seizures. Death is a possibility if the condition is left untreated. Major withdrawal symptoms (convulsions and delirium) may occur within 16 hours and last up to 5 days after abrupt cessation of these drugs. -
Question 77 of 100
77. Question
1 pointsA 50 year old female who has had low back pain for several years is admitted to the hospital because the pain has suddenly worsened. She is currently taking oxycodone, amitriptyline, perphenazine, fluoxetine and trazodone. On physical examination, the patient is 10% below her ideal body weight, pupils are constricted and skin turgor is poor. She seems sluggish and her speech is slow. Results of neurologic examination and radiographs of the lumbosacral spine are normal. Which of the following medication you suspect as the cause of her mental symptoms?
Correct
Incorrect
Explanation:
The question gives you the classic sign of opiate use: constricted pupils. Prolonged use of opiates may also induce depression. Amitriptyline can cause weight gain, and does not cause pupil constriction. Fluoxetine may result in mild weight loss of two to five pounds, but does not cause pupil constriction. Neither perphenazine nor trazodone cause pupillary constriction. -
Question 78 of 100
78. Question
1 pointsThe beta blocker that would be most useful for lowering blood pressure while minimizing bradycardia is
Correct
Incorrect
Explanation:
Labetalol has beta and alpha-adrenergic blockade. Labetalol causes blood pressure to drop more significantly than heart rate compared to other beta blockers like propranolol (A), metoprolol (cardioselective) (C), or esmolol (IV infusion, short-acting) (D). -
Question 79 of 100
79. Question
1 pointsDiuretic that may be ototoxic is which one of the following?
Correct
Incorrect
Explanation:
Acetozolamide works at the proximal tubule and prevents HCO3- reabsorption, which can cause a metabolic acidosis (furosemide and thiazides may cause a metabolic alkalosis). Furosemide works at the ascending loop of Henle and may be ototoxic (the other choices are not commonly ototoxic). Thiazide diuretics work at the distal convoluted tubule and can cause hyperuricemia. Spironolactone is a K+ sparing diuretic that works at the distal tubule and collecting duct (the other diuretics are more likely to cause hypokalemia). -
Question 80 of 100
80. Question
1 pointsA 29 year old HIV positive male presents with a recurrent whitish film that seems to grow on his tongue and oral mucosa. A sample of the material scrapped from the lesions reveals Candida albicans. What is the mechanism of action of the drug of choice for this infection?
Correct
Incorrect
Explanation:
This is a case of oral thrush, an overgrowth of the oral cavity with Candida albicans, which is a normal flora organism that commonly causes problems in immunosuppressed patients. The drug of choice is topical nystatin; and its mechanism of action, like that of amphotericin B, is that it complexes with ergosterol in the primary membrane sterol of the fungi, and therefore causes the membrane to become leaky. Terbinafine blocks ergosterol synthesis. This agent is primarily used to treat dermatophytic infections, particularly tinea unguium.
An experimental antifungal agent, pneumocandin, inhibits synthesis of beta glucan. Griseofulvin interferes with mitotic spindle function. This agent is primarily used topically against dermatophytes, as it concentrates in the stratum corneum. Flucytosine interferes with thymidylate synthetase. This agent is used in the treatment of cryptococcosis. It is contraindicated in HIV patients, as it may cause bone marrow suppression in those patients. -
Question 81 of 100
81. Question
1 pointsTesting of a new antifungal medication is being conducted in phase I clinical trials. Examination of the drug´s pharmacokinetic properties reveals that the drug´s half life is 6 hours. If a continuous intravenous infusion of this drug were started on a research subject, how much time would it take to reach 75% of steady state?
Correct
Incorrect
Explanation:
The half- life (t1/2) of a drug is the time required for serum concentration to decrease by one half (50%) after absorption and distribution are complete. The rule of thumb is that the plasma concentration will reach 50% in one half live, 75% in two half lives, 87.5% in three half lives, etc., so that the difference between the current drug level and 100% halves with each half-life. It takes four to five half-lives to reach steady state. In this instance, it takes two half lives to reach 75%. The half life of this drug is 6 hours so two half lives is 12 hours. -
Question 82 of 100
82. Question
1 pointsA man presents to the ER with chest pain, diaphoresis, and “a funny racing sensation in her chest.” An ECG is performed, that shows atrial fibrillation. After initial stabilization, which of the following medication should be given?
Correct
Incorrect
Explanation:
Atrial fibrillation occurs when small cell groups (up to thousands) of atrial muscle act independently of others. There are no detectable P waves because there is no synchronous depolarization, and action potentials arrive at the AV node at random intervals, leading to an erratic ventricular rhythm with fluctuating P-P intervals (irregularly Irregular). Patients are initially stabilized (usually with Diltiazem), and then rate control, chronic arrhythmia, and anticoagulation must be addressed. This question addresses the issue of anticoagulation in atrial fibrillation. The altered flow in the fibrillating atrium can cause clot formation, and thus anticoagulation must be initiated in order to prevent stroke or other embolic disease. Thus Warfarin is the correct choice. Atropine is a muscarinic cholinergic antagonist and would speed electrical conduction. This would exacerbate the problem. Morphine is incorrect. Once the patient is stabilized, and rate controlled, the patient´s pain should subside. There is thus no need for analgesia. Penicillin is effective against many bacteria, but thus there is no indication for it in this case. Propylthiouracil is used in the treatment of hyperthyroidism. It inhibits thyroid peroxidase and also diminishes the peripheral deiodination of T4 to T3. Hyperthyroidism can lead to atrial fibrillation, but there is nothing to indicate that the patient´s atrial fibrillation is due to hyperthyroidism. In either case, the patient needs anticoagulation first, so Warfarin is a better choice. -
Question 83 of 100
83. Question
1 pointsA 65 year old male is diagnosed with chronic bacterial prostatitis and urosepsis. He is treated with large doses of broad spectrum antibiotics for an extended period of time. What complication would most likely develop?
Correct
Incorrect
Explanation:
To answer this question you have to identify two pieces of information. First, you have to recognize that the question is about vitamin deficiency acquired by antibiotic therapy (vitamin K is made by bacteria in the gut), and then you must recognize the deficiency syndrome that would be produced (bleeding tendency secondary to the inability to make clotting factors II, VII, IX, X, and proteins C and S). The other vitamin/syndrome associations are as follows:
Vitamin D deficiency can lead to bony abnormalities. Vitamin A deficiency can result in decreased night vision. Vitamin B12 and thiamine deficiency can lead to neurologic defects. Vitamin C deficiency can lead to scurvy. -
Question 84 of 100
84. Question
1 pointsA 48 year old patient with coronary artery disease undergoes surgery to insert a stent. To reduce the risk of an adverse cardiovascular disease, a combination of aspirin and a second medication that has the same Antiplatelet indications as aspirin is given. If this second medication is associated with rare cases of severe bone-marrow toxicity, which statement best describes this medication´s mechanism of action?
Correct
Incorrect
Explanation:
Similar to clopidogrel, ticlopidine irreversibly blocks ADP from binding platelets. Subsequently, ADP induced expression of glycoprotein receptors is reduced, resulting in less fibrinogen binding. Because aspirin and ticlopidine work through distinct mechanisms, they typically synergize. Ticlopidine may be used as a second line agent to prevent thromboembolism or stroke. Rare cases of severe bone marrow toxicity limit its use to patients who are intolerant of or unresponsive to aspirin. The heparins, including enoxaparin and dalteparin, potentiate antithrombin III. The thrombolytic agent alteplase converts plasminogen to plasmin. Inhibition of vitamin K function is descriptive of Warfarin. Three potent parenteral GpIb/IIIa inhibitors (disintegrins) are used in the settings of percutaneous coronary intervention (PCI), unstable angina, and non Q wave MI: abciximab (c7E3 Fab), eptifibatide, and tirofiban. -
Question 85 of 100
85. Question
1 pointsA girl with a nonproductive cough, low grade fever, headache and noted to have a non-purulent otitis media is given an antibiotic that inhibits translocation of the growing peptide chain along the mRNA. Which antibiotic was prescribed?
Correct
Incorrect
Explanation:
This patient was infected with M. pneumoniae the most common cause of pneumonia in young adults. The disorder is usually characterized by a nonproductive cough, low grade fever, and a headache as well as nonpurulent otitis media or bullous myringitis in about 20% of the patients. It is usually treated with erythromycin, azithromycin, tetracyclines, or fluoroquinolones. Erythromycin acts to inhibit the translocation step of ribosomal protein synthesis. Chloramphenicol inhibits ribosomal peptidyl transferase in prokaryotes. Cycloheximide inhibits ribosomal peptidyl transferase in eukaryotes. Streptomycin causes misreading of the code during initiation in prokaryotes. Tetracycline acts on prokaryotes to prevent binding of the aminoacyl-tRNA to the ribosome, thereby inhibiting initiation. -
Question 86 of 100
86. Question
1 pointsA 48 years old man is brought to the ER by his wife after obtaining a blood pressure reading of 234/142 mm Hg on a home blood pressure monitor. On arrival his blood pressure is 236/144 mm Hg. He has always had poorly controlled blood pressure. He also states he took medication for a migraine headache 2 hours earlier. He most likely took which medication?
Correct
Incorrect
Explanation:
Hypertensive emergencies require substantial reduction of blood pressure within 1 hour of onset to lower the risk of serious morbidity or death. By definition, a hypertensive emergency is defined as a strikingly elevated diastolic blood pressure (>130 mm Hg).With respect to sumatriptan, a 5-HT1D agonist, significant elevations in blood pressure, including hypertensive crises, have been reported on rare occasions. The incidence of developing a hypertensive emergency is highest in patients with poorly controlled or uncontrolled blood pressure, hence, this medication is contraindicated in these patients. None of the other medications has been associated with the development of a hypertensive emergency. Diclofenac potassium is a rapid acting nonsteroidal anti-inflammatory drug indicated for the treatment of pain secondary to inflammation. This medication is primarily associated with gastrointestinal discomfort. Ergotamine has partial agonist/antagonist activity at the tryptaminergic, dopaminergic, and alpha adrenergic receptors. It is indicated for the treatment and prevention of vascular headaches and causes nausea and vomiting. Methysergide blocks the action of serotonin. It is indicated for the treatment and prevention of vascular headaches and can cause nausea and vomiting. Propranolol is a nonselective beta receptor blocking agent indicated for the treatment of hypertension and the prevention of migraine headaches. This agent is primarily associated with the development of bradycardia. -
Question 87 of 100
87. Question
1 pointsA patient is given an immunosuppressive drug that inhibits de novo purine synthesis and is myelosuppressive. Later dose reduction is needed because he is also taking Allopurinol. The drug likely used is
Correct
Incorrect
Explanation:
Azathioprine is converted to 6-mercaptopurine, which inhibits DNA synthesis and subsequent cell proliferation. Xanthine oxidase is instrumental in the metabolism of azathioprine metabolites, thus co-administration with allopurinol (an inhibitor of xanthine oxidase) can result in azathioprine toxicity.
Abatacept is as selective costimulation modulator that inhibits T-cell activation and is indicated for rheumatoid arthritis.
Cyclosporine and tacrolimus inhibit calcineurin mediated transcription of interleukin production, which reduces subsequent T-cell activation. Mycophenolate mofetil is converted to the active metabolite mycophenolic acid, which inhibits de novo guanine nucleotide synthesis and subsequent lymphocyte proliferation, these agents are both indicated for organ transplants but are not as likely to be affected by allopurinol. -
Question 88 of 100
88. Question
1 pointsOn examination of a 77 year old man the blood pressure is found to be 200/100 mm Hg. He has history of coronary artery disease, hypercholesterolemia, and a coronary artery bypass graft surgery 2 years ago. Which drug is contraindicated in managing his hypertension?
Correct
Incorrect
Explanation:
Medications for hypertension are targeted at the four factors that affect systemic blood pressure, including heart rate, cardiac contractility, peripheral vascular resistance and volume status. However, in patients who are bradycardic, or who have an extensive cardiac history and likely congestive heart failure, the use of medications that can reduce cardiac contractility or heart rate is contraindicated. Thus, beta blockers such as propanolol should not be used in the patient described in this question. Enalapril is an ACE inhibitor that works primarily by preventing the conversion of angiotensin I to angiotensin II, increasing the secretion of aldosterone. The effect of enalapril on the heart is less than that of beta blockers. Furosemide is a loop diuretic, which is an effective treatment for hypertension by reducing renal fluid retention and the body´s fluid load. Hydralazine is a direct acting vasodilator that reduces peripheral vascular resistance via the nitric oxide pathway. Losartan is an angiotensin receptor antagonist that acts in a similar fashion to ACE inhibitors, and primarily reduces peripheral vascular resistance and body fluid status, with little effect on the heart.
-
Question 89 of 100
89. Question
1 pointsA boy aged 14 years has been diagnosed with moderate persistent asthma. Recently his physician has decided to add an agent to the boy´s drug regimen that binds to free IgE. Which agent was likely prescribed?
Correct
Incorrect
Explanation:
Omalizumab is a monoclonal antibody that binds to human IgE. It inhibits the binding of IgE to the high-affinity Fc receptor on the surface of mast cells and basophils, thus limiting the release of mediators. It is indicated in patients who are 12 years of age or older, with moderate to severe persistent asthma, whose symptoms are not adequately controlled with corticosteroids. It is administered subcutaneously every 2-4 weeks. Beclomethasone is glucocorticoid used in asthma. It blocks phospholipase A2 and inhibits cyclooxygenase-2 (COX-2). Cromolyn and nedocromil inhibit the release of mediators from mast cells via a poorly understood mechanism. Infliximab is a monoclonal antibody that binds tumor necrosis factor-alpha (TNF-α) and is used in rheumatoid arthritis and Crohn disease. Ipratropium is a muscarinic antagonist with minimal systemic side effects because it is a quaternary amine. It is also used in asthma and is especially effective in asthmatics with COPD -
Question 90 of 100
90. Question
1 pointsAfter a MI, a 53 year old male develops a life threatening ventricular arrhythmia and is treated with IV Lidocaine. He is then switched to an oral agent with mechanism of action similar to lidocaine. Which drug was likely prescribed?
Correct
Incorrect
Explanation:
Lidocaine is a class IB antiarrhythmic indicated for the treatment of ventricular tachycardia. This agent blocks sodium channels and shortens the action potential duration. Both mexiletine and tocainide are classified as class IB antiarrhythmic agents that shorten the action potential duration and retractor; period and improve the resting potential duration. These agents produce a modest suppression of sinus node automaticity, as well as AV node conduction.
Esmolol is a class II antiarrhythmic agent that exerts its action as an ultra short acting beta adrenergic blocking agent with cardioselective properties. Diltiazem and verapamil are calcium channel blocking agents and class IV antiarrhythmic agents. As antiarrhythmic, these agents decrease and slow SA and AV node conduction. Both disopyramide and procainamide are class IA antiarrhythmic. They decrease myocardial excitability, conduction velocity, contractility, and automaticity. They also prolong the effective retractors period and block vagal stimulation of the AV node. -
Question 91 of 100
91. Question
1 pointsA 35 year old man with IV opioid drug abuse presents complaining of arm pain. He is mildly febrile, and a tender, erythematous, fluctuant mass is observed on the patient´s right arm. The area is prepped and draped and copiously infiltrated with 2% lidocaine. When an incision is made to drain the abscess, the patient complains of severe pain. The failure of the local anesthesia is explained by which one of the following?
Correct
Incorrect
Explanation:
Lidocaine is a common local anesthetic agent used in a variety of clinical situations from laceration repair to abscess drainage. In this case this patient has an abscess likely from using unclean drug paraphernalia. The areas of inflammation tend to have a lower pH than healthy tissue. Local anesthetics are weak bases and are only effective at penetrating tissue in the unprotonated form. Thus in acidic areas such as this abscess, lidocaine becomes protonated and poor anesthesia results. Chronic opioid abuse does not affect either the structure or regulation of the Na channels blocked by local anesthetics. Chronic opioid abuse may downregulate opioid receptors, but these are not involved in the mechanism of local anesthesia. While the edema fluid may slightly dilute the anesthetic the effect would be slight. The fact that lidocaine is an amide should have no bearing on its efficacy. -
Question 92 of 100
92. Question
1 pointsA medical student is performing experiments on an anesthetized animal for her pharmacology lab. An arterial line is inserted to monitor blood pressure, and the animal is given an intravenous dose of epinephrine. The injection produces an increase in blood pressure. She then injects an unknown drug, followed fifteen minutes later by re-administration of epinephrine. The second administration of epinephrine now produces a decrease in blood pressure. The unknown drug belong to which class?
Correct
Incorrect
Explanation:
This classic drug response is called epinephrine reversal. Epinephrine, a nonselective alpha and beta adrenergic agonist, increases blood pressure. The unknown drug is an alpha adrenergic antagonist, such as phentolamine, which blocks epinephrine´s vasoconstrictive action on arterioles. Subsequent administration of epinephrine produces only beta receptor stimulation, causing vasodilation in skeletal muscle, leading to a decrease in blood pressure. Epinephrine, for all practical purposes, now acts like the nonspecific beta agonist, isoproterenol. This effect is called epinephrine reversal because of the fact that epinephrine originally increases BP and then produces the opposite effect alter phentolamine administration. An acetylcholinesterase inhibitor should not affect the subsequent administration of epinephrine. A nicotinic ganglionic blocker may prevent a potential decrease in heart rate due to baroreceptor reflexes, but epinephrine would still cause an increase in blood pressure because its access to end organ receptors would be unaltered. A nonselective alpha agonist might not affect a second administration of epinephrine fifteen minutes later because the agonist effect would probably be gone. But, if there was still some agonist on board at the time of the second administration, it would only serve to enhance epinephrine´s increase in blood pressure. A nonselective beta receptor antagonist would enhance epinephrine´s increase in blood pressure. After administration of a beta antagonist such as propranolol, epinephrine would only produce alpha receptor stimulation. This would increase blood pressure to a greater extent than epinephrine alone. -
Question 93 of 100
93. Question
1 pointsA 23 year old man undergoes general anesthesia an inguinal hernia repair. Fentanyl, propofol, and midazolam are administered to achieve anesthesia. The surgeon quickly finishes the case, but the patient received too much fentanyl to breathe without assistance. The respiratory depression seen in this patient would be counteracted by which one of the following?
Correct
Incorrect
Explanation:
Fentanyl, an opioid analgesic, is frequently used during general anesthesia to produce analgesia and respiratory depression. While allowing the anesthesiologist to easily ventilate the patient intraoperatively, the respiratory depression produced by opioids may linger postoperatively. Patients will be difficult to extubate, as their central respiratory drive is suppressed. Administering naloxone, an opioid antagonist, may expedite this process. Care should be taken when using naloxone: administering too much will not only counteract the respiratory depression, it will also counteract the analgesic properties. Flumazenil is used to treat benzodiazepine overdose. Benzodiazepines contribute to respiratory depression, but the question identities fentanyl as the source of the patient´s respiratory depression. Protamine is used to reverse heparinization. It is not used to treat respiratory depression. Hyperventilation followed by extubation will not reverse the respiratory depression. This treatment may further diminish the respiratory drive by further lowering CO2. Positioning the patient in a head-up position will not reduce the respiratory depression associated with narcotics. -
Question 94 of 100
94. Question
1 pointsA 67 year old female develops congestive heart failure secondary to left ventricular dysfunction. After discharge she is prescribed indapamide as part of her congestive heart failure treatment regimen. Which substance excretion would be decreased by this agent?
Correct
Incorrect
Explanation:
Indapamide is classified as a thiazide diuretic. Thiazide diuretics, such as indapamide, hydrochlorothiazide, and metolazone, are commonly used to treat various forms of edema and essential hypertension. One of the unlabeled uses of this medication is for the adjunctive treatment/prevention of osteoporosis in postmenopausal women with hypertension. Thiazides are effective in reducing the risk of osteoporosis because they decrease the elimination of calcium. Indapamide, like other thiazide diuretics, increases the elimination of chloride, magnesium, potassium, and sodium. Because potassium loss can be substantial, patients are generally advised to take this medication with food products high in potassium, such as bananas or orange juice. -
Question 95 of 100
95. Question
1 pointsA male aged 72 years is seen for a routine examination. His BP is 170/100 mm Hg. His physician prescribes Amlodipine. This agent exerts its mechanism of action by blocking which one of the following?
Correct
Incorrect
Explanation:
Amlodipine is calcium channel blocking agent indicated for the treatment of mild to moderate hypertension and angina. Amlodipine selectively blocks calcium ion influx across the membranes of cardiac and vascular smooth muscle cells without changing serum calcium concentrations. Terazosin is an example of an alpha 1 adrenergic receptor-blocking agent that is indicated for the treatment of hypertension and benign prostatic hypertrophy. Enalapril is an example of an angiotensin-converting enzyme (ACE) inhibitor indicated for the treatment of hypertension and congestive heart failure. When ACE is inhibited, it prevents the conversion of angiotensin I to angiotensin II, leading to decrease in aldosterone levels.
Propranolol is an example of a non-selective beta 1 receptor and beta-2 receptor blocking agent indicated for the treatment of hypertension. Beta-1 receptor blockade leads to negative inotropic and chronotropic effects on the heart. Beta-2 receptor blockade causes bronchoconstriction. -
Question 96 of 100
96. Question
1 pointsA 59 year old female arrives at her clinician´s office complaining of moderate anxiety. The drug that will help relieve her anxiety, with a minimum of unwanted sedative side effects is which one of the following?
Correct
Incorrect
Explanation:
Buspirone is a nonbenzodiazepine anxiolytic that is devoid of the sedative (or anticonvulsive and muscle relaxant) properties typically associated with the benzodiazepines. It is a partial agonist at 5-HT1A receptors.
Chlordiazepoxide and lorazepam are benzodiazepines. Although they are useful anxiolytics, they produce sedation. Trazodone is a very sedating atypical antidepressant. Zolpidem is a nonbenzodiazepine hypnotic used for the treatment of insomnia. -
Question 97 of 100
97. Question
1 pointsA male aged 34 years with a long history of asthma is referred to a pulmonologist. The physician decides to prescribe zileuton. Which one of the following is the mechanism of action of this drug?
Correct
Incorrect
Explanation:
Zileuton is a recently approved oral inhibitor of 5-lipoxygenase, the first enzyme in the pathway from arachidonic acid to leukotrienes. Leukotrienes are synthesized in many inflammatory cells in the airways, such as mast cells, macrophages, eosinophils, and basophils. LTC4 and LTD4 are thought to be responsible for many of the symptoms of asthma, including bronchoconstriction, increased bronchial reactivity, hypersecretion of mucus, and mucosal edema. In addition, LTB4 is a potent chemotactic agent for neutrophils. Zileuton and similar agents are efficacious in the treatment of asthma because they inhibit leukotriene production. Zafirlukast is another drug that is used to interrupt the leukotriene pathway. This drug acts as an LTD4 antagonist; it is taken orally.
Methylxanthines, such as theophylline, inhibit phosphodiesterase, thus increasing intracellular levels of cAMP and resulting in smooth muscle relaxation. At therapeutic doses, methylxanthines also block adenosine receptors. Corticosteroids prevent the release of arachidonic acid from cell membranes by inhibiting phospholipase A2. This reduces the production of both leukotrienes and prostaglandin. Corticosteroids also inhibit the production of cytokines, which are thought to play an important role in initiating the inflammatory cascade provoked by antigen inhalation and viral infection. Examples of corticosteroids include beclomethasone, budesonide, flunisolide, fluticasone, and triamcinolone. Beta2 agonists, such as albuterol, terbutaline, metaproterenol, and bitolterol, cause smooth muscle relaxation by increasing intracellular levels of CAMP. -
Question 98 of 100
98. Question
1 pointsA professor is teaching his class about the actions of different drugs on vessels. A certain drug produces vasodilation by increasing cGMP in the smooth muscle cells of arterioles. The drug that has this mechanism of action is
Correct
Incorrect
Explanation:
Muscarinic agonists stimulate noninnervated muscarinic (M3) receptors located on the endothelium of blood vessels. These receptors activate phospholipase C (via G q), thereby raising levels of inositol triphosphate (IP3 and diacylglycerol (DAG). IP3 produces an increase in intracellular calcium, which activates nitric oxide synthase and thus produces nitric oxide. Nitric oxide, in turn, diffuses to the smooth muscle cells of arterioles and raises cGMP by activating guanylate cyclase. cGMP causes vasodilatation, by dephosphorylating myosin light chain, thus preventing the interaction of myosin with actin. Isoproterenol, a nonselective beta agonist, causes vasodilatation by activating beta2 receptors located in the smooth muscle of arterioles. Beta2 receptors activate adenylate cyclase, thus raising levels of cAMP. cAMP activates cAMP-dependent protein kinase (protein kinase A), which phosphorylates myosin light chain kinase (MLCK), rendering this enzyme inactive. MLCK can no longer phosphorylate the myosin light chain, an essential step in smooth muscle contraction. Metaproterenol, a beta2 agonist, causes vasodilatation by the same mechanism as isoproterenol. Nifedipine blocks L-type calcium channels in both smooth and cardiac muscle. Increased blocking of calcium channels in the smooth muscle cells of arterioles prevents the activation of MLCK. MLCK can no longer phosphorylate the myosin light chain, an essential step in smooth muscle contraction. Phentolamine causes vasodilatation by blocking alpha receptors on arteriolar smooth muscle cells. Alpha1 receptors cause constriction by IP3-induced increases in intracellular calcium. Phentolamine therefore prevents the increase in intracellular calcium, thereby causing less activation of MLCK. -
Question 99 of 100
99. Question
1 pointsA person who was drug abuser in past presents to ask for pain medication for a legitimate back pain. The physician takes the history of drug abuse into account. Which medication has the greatest potential for abuse?
Correct
Incorrect
Explanation:
The drug that is most likely to be abused is that which causes the greatest euphoric effect. Euphoria is mediated by mu opioid receptors, and meperidine is the only drug listed that is a full mu agonist. Codeine is a moderately effective agonist at the mu receptor and therefore has less abuse potential than meperidine. Dextromethorphan is an over the counter antitussive agent with limited abuse potential. Loperamide is an over the counter antidiarrheal agent with very limited abuse potential. Nalbuphine is a mixed agonist/antagonist (also called a partial agonist) with antagonist actions at the mu receptor. As a result, it has less abuse potential than meperidine. -
Question 100 of 100
100. Question
1 pointsA 75 year old male has not been able to pass urine today, but had been able to do so normally the previous 2 days. Exam reveals a BP of 175/90 mm Hg. Lab data reveals creatinine of 4.5 mg/dL and BUN of 115 mg/dL. Urinalysis reveals a specific gravity of 1.01 mg/dL and an occasional white blood cell per high powered field. Which could be used to ameliorate his symptoms?
Correct
Incorrect
Explanation:
Prostatic hypertrophy in elderly men is very common; therefore, it should be considered as a primary cause of renal insufficiency until proven otherwise. The patient´s signs and symptoms are consistent with obstructive uropathy; there is a history of high urine output followed by periods of almost no urine output. This pattern leads to the accumulation of urine in the collecting system, which creates a high pressure system. The high pressure is then “transmitted” back to the kidney and results in renal insufficiency. Since the patient´s obstructive uropathy is most likely caused by prostatic hypertrophy, doxazosin should be used to treat the cause of these signs and symptoms. Doxazosin is a peripherally acting alpha1 adrenergic blocking agent indicated for the treatment of urinary outflow obstruction secondary to benign prostatic hyperplasia (BPH). It is also indicated for the treatment of hypertension, especially in men with BPH. Therefore, the use of this agent will correct the obstructive uropathy and treat his hypertension. Benazepril is an ACE inhibitor used in the treatment of hypertension; however, it is known to cause azotemia and oliguria, especially in those with renal insufficiency. Therefore, this agent would be contraindicated. Furosemide is a loop diuretic used to increase urine output in patients without a urinary tract obstruction. Hyoscyamine is used in the treatment of gastrointestinal disorders caused by spasm and hypermotility. Since this agent is a potent anticholinergic, it would not be recommended in a patient with urinary obstruction. Remember, anticholinergic agents cause urinary retention. Phenazopyridine is a urinary tract analgesic used to decrease the dysuria associated with urinary tract infections. The use of this agent in patients with renal insufficiency is not recommended because phenazopyridine can accumulate, resulting in renal stones and transient renal failure.